Download as pdf or txt
Download as pdf or txt
You are on page 1of 42

Prime Mock 18 (CLAT) 2024

English Language
Directions for questions 1 to 30: Each set of questions in this section is based on a single passage. Please answer
each question on the basis of what is stated or implied in the corresponding passage. In some instances, more than
one option may be the answer to the question; in such a case, please choose the option that most accurately and
comprehensively answers the question.

Passage – 1

Every morning when I wake up, I lean out my window to say hello to Mom. She doesn't reply, but that's okay. She
never was a good listener, even before she was buried in our backyard.

My only sibling, a little brother, Dill, plays there most afternoons. When other kids his age run to the playground after
school, shrieking and chasing each other like little monkeys, he always comes straight home to me. He's a loner like
his sister, and I'm a little bit proud of that.

Wildflowers sprout from the bumpy soil where Mom sleeps. I'd like to picture them emerging from her eyes and hands
and heart; those flowers will be the only beautiful things Mom's given birth to other than Dylan-Dyl-Dill, but that image
is all wrong. I know for a fact she's buried face down. The closest thing to a flower is her caved-in skull. The blood
blossomed there like nothing I'd ever seen.

Dill doesn't know he's walking above his mother's body, just as she used to trample all over us. I sit on the back porch
and watch him care for his worm farm. The little writhe creatures flourish under his five-year-old fingers. Sometimes I
like to imagine he tells those worms secrets, and they crawl to Mom's empty ears, whispering them back.

She's jealous when she learns how happy we've been without her. The worms say I'm much better than she'd ever
been, anyway. I sign my brother's notes from school, tuck him in, feed him his greens, Dill Pickle, here comes the
aeroplane.

Dill runs up to me. I ruffle his hair and give him a glass of freshly-squeezed lemonade. He sits on my lap and drinks it
while we look out at our backyard.

I ask him, "Aren't we happy?"


He nods his head with vigour.
The worms nod, too.
Q 1. 30630558  The author's brother nods his head with 'vigour'. Select the closest meaning of this term.

a)  Nod head lazily b)  Nod head with hesitation c)   d)  Nod head with energy
Nod head with acceptance

Q 2. 30630558  According to the given passage, what was the reason behind the death of the author' mother?

a)  The author's mother died due to a heavy blow on her head.

b)  The author's mother was killed by the worms who fed on her unconscious body.
c)  The author's mother has not died and lives in the cave.

d)  Insufficient information

Q 3. 30630558  The information given in the passage indicates which of the following?

a)  The gender of the author is female.

b)  The author mourns regularly for her mother's death.


c)  The author has a problem that is signified by her action every morning.

d)  None of the above is true

Q 4. 30630558  What is the author suggesting by pointing out that worms nod?

a)  The worms nod by seeing the two siblings enjoy.

b)  The author signifies that probably her mother is happy at seeing her children.
c)  The worms are happy that they are able to feed on the mother of the author.

d)  The author is pointlessly trying to suggest that her mother is alive.

Q 5. 30630558  What is the correct meaning of the term 'trample'?

a)   b)  
To walk over someone/something without hurting To walk on somebody/something and damage or hurt
him/her/it. him/her/it

c)  To teach someone a life lesson d)  To take revenge for a past misconduct

Directions for questions 1 to 30: Each set of questions in this section is based on a single passage. Please answer
each question on the basis of what is stated or implied in the corresponding passage. In some instances, more than
one option may be the answer to the question; in such a case, please choose the option that most accurately and
comprehensively answers the question.

Passage – 2

India has been in the grip of what seems like an eternity of heat waves. April temperatures over north-west and
central India are the highest in 122 years. During April 1 to 28, the average monthly maximum temperature over
northwest India was 35.9° Celsius and the same over central India was 37.78° C. These averages belie
measurements at the district and sub-divisional level where several parts of Rajasthan, Punjab, Haryana, Gujarat,
and Maharashtra have seen temperatures inch towards the mid-40s and breach normals. There is little respite
expected in May, which is anyway the hottest month, though the India Meteorological Department (IMD) says that
while north and west India will continue to sizzle on expected lines, and must likely brace for more heat waves, the
rest of the country is unlikely to see the levels of March and April. The proximate causes for the searing heat are an
absence of rain-bearing Western Disturbances, or tropical storms that bring rain from the Mediterranean over north
India. Cool temperatures in the central Pacific, or a La Niña, that normally aid rain in India, too have failed to bolster
rainfall this year. This is an unusual occurrence.

Despite five Western Disturbances forming in April, none was strong enough to bring significant rain and depress
temperatures. The IMD has forecast a 'normal' monsoon or 99% of the Long Period Average (LPA) of 87 cm and is
expected to forecast the monsoon's arrival over Kerala later in May. On the surface, there is no direct bearing
between the intensity of heat waves and the arrival and performance of the monsoon. While individual weather events
cannot be linked to greenhouse gas levels, a warming globe means increased instances of extreme rain events and
extended rain-less spells. What is better known is it helps to have disaster management plans in place that help
States better deal with heat waves and their impact on health. The official toll due to heat waves in the last 50 years is
put at over 17,000 people, according to research from the IMD. The heat island effect means urbanisation adds
degrees to the already searing conditions; and so, heat wave deaths must be treated as a disaster that merits
compensation. Private and public workplaces too must be better equipped to factor heat wave risk.
Q 6. 30630558  What is the central purpose of the author in the passage?

a)   b)  
To encourage people to take precautions against heat To highlight the 'disaster' that is caused due to heat wave
wave c)  
To explain how the present heat wave is a man-made
disaster

d)  
To reason out the instances of extreme heat waves in
India

Q 7. 30630558  Based on the passage, it can be said that

a)  Extreme heat waves in the country should make the authorities work on disaster management plans related to it.

b)  The month of May would bring in respite from the heat waves for most parts of India.

c)  The kinds of heat waves observed in the month of April have not been witnessed before in India.

d)  All of the above

Q 8. 30630558  Why does the author think that heat wave deaths merit compensation?

a)  Because the heat waves are induced completely by the actions of humans

b)  Because heat wave deaths merit compensation on moral grounds

c)  Because urbanisation plays a role in exaggerating heat waves

d)  Because it is the job of the state to compensate the kin of the deceased

Q 9. 30630558  What is the correct interpretation of the term 'bearing' as used in the passage?

a)  Friction (Physics) b)  Enmity c)  Relationship d)  Networking

Q 10. 30630558  The author of the passage would not agree with which of the following?

a)  Most parts of India would see relief from the heat waves in May.

b)  La Niña disrupts rainfall in India.

c)  Western Disturbances can tackle the ill-effects of all heat waves at once.

d)  All of the above

Directions for questions 1 to 30: Each set of questions in this section is based on a single passage. Please answer
each question on the basis of what is stated or implied in the corresponding passage. In some instances, more than
one option may be the answer to the question; in such a case, please choose the option that most accurately and
comprehensively answers the question.

Passage – 3

Prime Minister Imran Khan showed on Sunday that the passionate base of his Pakistan Tehreek-e-Insaf (PTI)
remained intact when tens of thousands of people turned up at Islamabad's Parade Ground to attend his rally. But his
long speech also indicated that the cricketer-turned-politician has finally come to terms with the political challenges he
is facing. Mr. Khan, who is facing a no-confidence vote in Parliament, used his carefully worded speech to defend the
performance of his government, reiterate the PTI's "Islamic welfarist" ideology, and set the tone for the future political
battles. Drawing parallels between himself and Zulfikar Ali Bhutto, the former Prime Minister who was overthrown in
1977 by General Zia-ul-Haq and later executed, Mr. Khan alleged that there was an international conspiracy against
his government. Whether his allegations are true or not, his coalition government is in trouble, surviving on a razor-
thin majority in Parliament. At least a dozen lawmakers from his party have revolted against him. Some coalition
members have hinted that there could be an early election. Mr. Khan's invoking of Bhutto's fate is also seen as veiled
criticism of the military establishment. He had been a close ally of the Generals from before the 2018 elections. But
the establishment appears to have gone cold on him.

While Mr. Khan has fiercely defended the track record of his government, all is not well at the ground level. There is
widespread resentment against the government's handling of the economy, which the Opposition has tried to capture
by mobilising support for the no-trust motion. The crisis is so deep that even government officials are reportedly not
paid their salaries on time. On Sunday, Mr. Khan said he would continue his crusade against "white collar crimes",
referring to corruption charges against Opposition leaders. He calls his opponents rodents and has brought dozens of
corruption cases against them in the past four years, but none has resulted in a conviction. On the other side, his
hardline approach has galvanized the Opposition, including the Pakistan Muslim League-Nawaz and the Pakistan
People's Party, which joined hands with Maulana Fazal-ur-Rehman of the Jamiat Ulema-e-Islam to form the Pakistan
Democratic Movement, an umbrella organisation whose sole goal is to bring the PTI government down. While crises
piled up one after another, Mr. Khan's approval rating started slipping. According to a Gallup poll in January, Nawaz
Sharif, currently living in exile, was 19 points ahead of Mr. Khan in popularity. Whether Mr. Khan survives the no-trust
vote or not, his run as a clean anti-corruption crusader with a mission to build 'Naya Pakistan' has come to a halt. His
political survival would depend on how he is going to adapt himself and the PTI to the new political environment
where he faces a united Opposition without the direct assistance of the military establishment.
Q 11. 30630558  Which of the following is untrue as per the passage?

a)  Prime Minister's long speech indicated that he has finally come to terms with the political challenges.

b)  The PM had failed to defend the track record of his government in the past two and half years.

c)  The PM's run as a clean anti-corruption crusader with a mission to build 'Naya Pakistan' has come to a halt.

d)  At least a dozen lawmakers from the PM's party have revolted against him.

Q 12. 30630558  Which of the following best describes the meaning of the word 'crusade' as per the passage?

a)  To fight for a cause b)  To adhere to rules c)  Be a voice for minority d)  To do what's right

Q 13. 30630558  Which of the following is the defense of PM Imran Khan for a no-confidence vote in Parliament?

a)  That there was an international conspiracy against his government.

b)  That at least a dozen lawmakers from his party have revolted against him

c)  That he had been a close ally of the Generals from before the 2018 elections, but they have betrayed him.

d)  That there is widespread resentment against the government's handling of the economy, which is wrong.

Q 14. 30630558  Which of the following best describes the writing style of the author in the passage?

a)  Satirical b)  Indignant c)  Acerbic d)  Maudlin

Q 15. 30630558  Why the PM has drawn a parallel between himself and Zulfikar Ali Bhutto?
a)  Because even Zulfikar Ali Bhutto faced a no-confidence motion in parliament.

b)  Because even Bhutto lost the confidence of the military.

c)  Because even Bhutto faced an international conspiracy against his government.

d)  All of the above

Directions for questions 1 to 30: Each set of questions in this section is based on a single passage. Please answer
each question on the basis of what is stated or implied in the corresponding passage. In some instances, more than
one option may be the answer to the question; in such a case, please choose the option that most accurately and
comprehensively answers the question.

Passage – 4

A one-of-a-kind gold neck ring from the Germanic Iron Age (400-550 A.D.) has been discovered near Ilsted in
southern Denmark. The necklace is heavy at 446 grams, a fraction under a pound, and is eight inches wide at the
widest point. It is made of one long rod-shaped piece of gold folded over itself at the terminals to create a ring shape.
The ends overlap about 1/3rd of the length of the necklace, and a gold plate is soldered to the back of the two rings to
create a third canvas for decoration between the two rings.

The overlapping ends of the rod are decorated with crescent-shaped depression stamped into the gold. The
decoration is so meticulously detailed that the crescent shapes on the two rings are ever so slightly different: the
crescents on the outer ring have eight decorative divots inside them, the crescents on the inner ones have six. The
gold plate has six ribbed gold threads at the bottom, plaited together two-by-two to create a chevron effect. A spiral
twisted gold wire runs down the middle of the plait.

Only ten comparable gold necklaces with stamped embellishment have been found in Denmark, and this is by far the
most elaborate, most finely worked of them. It is the only one with a soldered plate with intricate gold thread
decoration.

It was discovered by metal detectorist Dan Christensen in October 2021. Christensen works as an archaeological
scout for the Southwest Jutland Museums, so when he found the neck ring, he immediately alerted museum staff. In
the week following the discovery, the entire field was scanned with metal detectors in case the ring had been part of a
larger grouping of precious objects scattered by agricultural activity. (Previous examples of neck rings from this period
have been found in pairs.) Nothing turned up.

A subsequent full excavation of the find site revealed evidence of a settlement under a thin layer of plow soil,
including roof-bearing post holes from multiple three-nave longhouses dating to between 300 and 600 A.D. The neck
ring was found inside one of the longhouses. It was recovered from below the plow layer, so archaeologists believe it
was buried where it was found.

This is an unusual context for gold neck rings, as most of them have been found in wetlands where they were
deposited as votive offerings to the gods. The find site is on a promontory surrounded by bogs on three sides, so the
fact that this necklace was buried inside a longhouse when wetlands were available a few steps away in every
direction suggest it was being deliberately hidden to keep it safe during a period of danger or unrest but the owner
was never able to retrieve it.

Q 16. 30630558  Which of these is not a true statement about the necklace?


a)  The necklace weighs below 500 grams.

b)  It might be somewhere below 8 inches in width at the slimmest point.

c)  It is not made up of different small pieces of gold.

d)  A lot of gold has been wasted in joining several pieces of gold in this necklace.

Q 17. 30630558  It can be inferred from the passage that:

a)  The Germanic Iron Age primarily used only iron and gold to make their tools and equipments.

b)  Ancient civilization existed in parts of Scandinavia more than a thousand years back.

c)  Ancient civilization existed in parts of Scandinavia even before the birth of Christ.

d)  
The civilization that existed in Denmark during the Germanic Iron Age was the most advanced European civilization in
the entire Europe.

Q 18. 30630558  Which of the following attribute of the author is highlighted in the last paragraph?

a)  Strong hold over facts. b)  Good sense of logical deduction.

c)  A fine balance of realism and optimism. d)  Inclination towards irrelevant facts.

Q 19. 30630558  Which of the following is not a synonym of 'crescent'?

a)  Demilune b)  Arc c)  Crook d)  None of these

Q 20. 30630558  Why did the team of archaeologists scan the entire field, after getting this particular necklace?

a)  Since it was assumed that this must be a subset.

b)  Since the necklace had some missing parts.

c)  Since it was assumed that the necklace must be broken into pieces.
d)  Since the necklace indicated presence of some human being.

Directions for questions 1 to 30: Each set of questions in this section is based on a single passage. Please answer
each question on the basis of what is stated or implied in the corresponding passage. In some instances, more than
one option may be the answer to the question; in such a case, please choose the option that most accurately and
comprehensively answers the question.

Passage – 5

Lula was Brazil's president for two terms from 2003 to 2006 and then 2007 to 2011, during which period the country
saw a massive commodities boom, which paid for sweeping social welfare programmes that are credited with having
raised millions of people out of poverty.

He is a charismatic leader with a frequently-recounted backstory: he could not read or write until he was 10 years old,
and he had to stop studying after grade 5 because he needed to work full-time. He became a metalworker and was
elected leader of the workers' union in 1975. He led large workers' strikes against Brazil's military regime, and in
1980, he co-founded the Workers' Party (PT) that would become the main leftwing political force in the country.

Lula entered Congress in 1986, and made several attempts at winning the presidency, a goal that he could finally
achieve in 2002. When he left office at the end of 2010 because Brazilian law does not allow a third consecutive
presidential term, he had an approval rating of 90 per cent.

The next decade was difficult for Lula, and tumultuous for Brazil. Soon after leaving office, Lula had to fight cancer of
the throat, and his successor in the post, Dilma Rousseff, was impeached and removed from office in 2016. The
following year, Lula was convicted of charges of corruption and money laundering in an investigation into the working
of the state-run oil giant Petrobras.

In April 2018, a year before Bolsonaro came to power, Lula started to serve a 12-year sentence in prison. However, in
March 2021, Lula's conviction was overturned by Brazil's Supreme Court, paving the way for his return to political life
after serving 580 days in prison.

India has had excellent relations with Brazil under Lula. He visited India as the Chief Guest at the Republic Day
parade in 2004, and then again in 2007 and 2008, and he hosted Prime Minister Manmohan Singh in 2006 and 2010,
and President Pratibha Patil in 2008. Lula was president of Brazil when the first BRIC (South Africa was still not a
member of the grouping then) was held in Russia in 2009.

The presidential election was a contest between Brazil's two biggest leaders, occupying two poles of the political
spectrum. President Bolsonaro had come to power on a far right platform, and had pressed forward with his agenda -
which has been seen to have resulted in mismanagement of the Covid-19 pandemic at home, and pushed the world
closer to environmental catastrophe by wanton destruction of the Amazon rainforest.

Over the past decade, Brazil's economy has slowed down considerably. While Bolsonaro had promised greater
deregulation and privatization, Lula has focused on food and housing for the poor, especially those who have been hit
the worst by the pandemic.

Lula's victory underlines the decisive shift to the left in Latin American politics. The change began with the election in
2018 of Andres Manuel Lopez Obrador in Mexico, and was followed by the victories for Alberto Fernandez in
Argentina in 2019, and Gabriel Boric in Chile, Pedro Castillo in Peru, and Gustavo Petro in Colombia. After Lula's
victory, six of the region's biggest economies will be run by Leftist politicians.

Still, the margin of Bolsonaro's defeat has been very thin, and he remains a powerful figure in the global surge of
right-wing populism. Before the election, there were fears, stoked by Bolsonaro's own statements, that he might, in
the case of defeat, seek to undermine the integrity of the electoral process or to reject its outcome altogether.
Q 21. 30630558  Which of the following is correct about Lula?

a)  Lula survived cancer and impeachment in succession.

b)  Lula was convicted of corruption and money laundering in the year Bolsonaro came to power

c)  Lula holds one of the extremes in the political spectrum.

d)  Lula endorses increased red-tapism and a crackdown on privatization.

Q 22. 30630558  What is overall tone of the author of the passage?

a)  Euphoric b)  Eulogy c)  Retrospective d)  Tactful

Q 23. 30630558  Which of the following can be termed as a suitable title of the passage?

a)  Lula and his wits b)  Politics of Lula c)  How Lula changed Brazil d)  How far has Lula come

Q 24. 30630558  What significance does Lula's victory hold in the Brazilian politics?
a)  Lula's victory underlines the decisive shift to the left in North American politics.

b)  With Lula now as the Brazilian president, six Latin American nations will be left-leaning.

c)  The presidential election, for the very first time, was a contest between Brazil's two biggest leaders.

d)  All of these.

Q 25. 30630558  Which of the following is farthest in meaning to the word 'Charismatic'?

a)  Charming b)  Fascinating c)  Magnetic d)  Preposterous

Directions for questions 1 to 30: Each set of questions in this section is based on a single passage. Please answer
each question on the basis of what is stated or implied in the corresponding passage. In some instances, more than
one option may be the answer to the question; in such a case, please choose the option that most accurately and
comprehensively answers the question.

Passage – 6

STATES depend on the consent of the governed: there are only a finite number of cops, and politicians, once stripped
of authority and armed guards, are about as liable to die from gunshot wounds or immolation as anyone else. The
deepest wish of the modern state is to forge precisely the sort of population which might perfectly support it.

The cruder versions of this practice are arithmetical. Fostering the right sort of births from the right sort of families, the
inclusion of the right sort of immigrants from acceptable countries: such enlargements are addition. Genocide, forced
disappearances, and summary executions represent subtraction. These are brutish tools of the state.

But the operation the state truly longs to master is not arithmetical but algorithmic. For the state, this is the
philosopher's stone, the perpetual motion machine, the good shit: not to increase or decrease a people but transform
it. Residential schools and re-education camps, de-radicalization and assimilation, nation-building and detention
centers--we could keep going. The dream of the state is to build a machine which could transmute one type of citizen
into another. In those halcyon days when Manhattan's towers still smoldered, some even believed that such machines
could be packed up for export and deployed, with suppressive fire, in the middle of Kabul or Baghdad or Pyongyang.

The trouble is that, despite decades of field trials, the machines just aren't that good. States, of all varieties, keep
feeding in people, but the results remain inconsistent at best. Sometimes it works, and model citizens are
manufactured. Other times, the people who emerge are just traumatized versions of those who entered. In the very
worst cases, the machines spew out embittered individuals who keep suspiciously glancing at critical national
infrastructure. There are entire peoples that each state struggles to digest, those of inauspicious nationalities, suspect
cultures, or doubtful pedigree: populations resistant to correction and regulation, grit in the machine.

But when the machine is fed children, it nearly always functions perfectly. The child of a subversive whistles the
national anthem on the way home from school. The immigrant parent finds their accent mocked by offspring who
dream only in the coarse tongue of their hostile new home. The foreign child is a perfect input, a blank slate to mold
into a citizen as easily as one might the native-born. The child of the most unbecoming parentage can become
cleansed, perfected, born anew. Such profound thaumaturgy is concealed behind words like "belonging," "inclusion,"
"charity," and "home." Any act of malfeasance required to facilitate a metamorphosis so sublime is surely forgivable: a
kidnapping never given the name, a mother never given a grave.
To build the machine is the dream of each contemporary state. To feed it the children of the world is the privilege of
empire.
Q 26. 30630558  With which of the following the author would least likely agree?

a)  A state is not independent of its subjects.

b)  Politicians are no different from general public.

c)  Machines can never be better than humans in governing a group.

d)  There is certainly a state of perfection in the functioning of the state.

Q 27. 30630558  Which of the following words is closest in meaning to the word 'embitter'?

a)  Anger b)  Merry c)  Elated d)  Boisterous

Q 28. 30630558  How does an immigrant child benefit the state in furthering its goals?

a)  An immigrant's child is least likely to get influence by the state.

b)  An immigrant's child helps in soothing people's anger.

c)  An immigrant's child is always easy to brainwash.

d)  All of these

Q 29. 30630558  Which of the following is not one of the ways the state employs the good shit?

a)  Inclusion of immigrants b)  Assimilation of the controlled subjects

c)  Forced disappearances d)  Summary executions

Q 30. 30630558  What can be deduced about the author's attitude towards the state?

a)  Hostile b)  Insincere c)  Admiring d)  Unbalanced

Current Affairs Including General Knowledge


Passage – 1

The World Health Organization ended the global emergency status for COVID-19 more than three years after its
original declaration, and said countries should now manage the virus that killed more than 6.9 million people along
with other infectious diseases.

The global health agency's Emergency Committee met and recommended the UN organization declare an end to the
coronavirus crisis as a "public health emergency of international concern" - its highest level of alert - which has been
in place since Jan. 30, 2020.

"It is therefore with great hope that I declare COVID-19 over as a global health emergency," said WHO Director-
General Tedros Adhanom Ghebreyesus, adding that the end of the emergency did not mean COVID was over as a
global health threat. During a lengthy conference call to brief the press on the decision, some WHO members became
emotional as they urged countries to reflect on lessons learned during the pandemic.

Ending the emergency could mean that international collaboration or funding efforts are also brought to an end or shift
in focus, although many have already adapted as the pandemic receded in different regions. "The battle is not over.
We still have weaknesses and those weaknesses that we still have in our system will be exposed by this virus or
another virus. And it needs to be fixed," said the WHO's emergencies director Michael Ryan.
Q 31. 30630558  What did the World Health Organization Director-General mention about COVID-19 in his
announcement?

a)  COVID-19 is no longer a global diseases. b)  COVID-19 has transformed into a seasonal outbreak.

c)  COVID-19 is over as a global health emergency. d)  COVID-19 is no longer a concern for the world.

Q 32. 30630558  Why did the World Health Organization (WHO) announce that mpox no longer constitutes a global
health emergency?

a)  The virus has been eradicated worldwide. b)  Cases of mpox have significantly decreased.
c)   d)  The symptoms of mpox have become less severe.
A new effective treatment for mpox has been developed.

Q 33. 30630558  What did experts with the World Health Organization (WHO) suggest regarding the handling of
COVID-19?

a)  It should be treated like a seasonal influenza. b)  It should be declared a global health threat.

c)  It should be categorized as a pandemic. d)  It should be ignored and not given much attention.

Q 34. 30630558  What is an outbreak?

a)  A decrease in disease cases compared to the expected number


b)  The occurrence of more disease cases than expected in a specific location and time.

c)  The spread of a disease globally

d)  A sudden decrease in disease transmission

Q 35. 30630558  What is an important aspect of addressing outbreaks and preventing further spread?

a)  Ensuring access to tests, vaccines, and treatments b)  Isolating affected individuals immediately

c)  Implementing travel restrictions d)  Providing health education to the general population

Q 36. 30630558  How is mpox transmitted to humans?

a)  Through the bite of infected mosquitoes. b)  By consuming contaminated food or water.
c)  Through direct contact with infected animals. d)  By inhaling airborne droplets from infected individuals.

Q 37. 30630558  What is mpox?

a)  A bacterial disease caused by an airborne pathogen.


b)  A viral disease similar to smallpox but less severe.

c)  A parasitic infection transmitted through contaminated water.

d)  A fungal infection affecting the respiratory system.

Passage – 2

Pakistan has descended into chaos. From a power struggle between the government and Imran Khan to a tussle
between the former Pakistan premier and the army to reports of infighting within the Pakistani military and a pull and
push between the judiciary and the civilian-military administration, there is a rumble in the Pakistan army. Pakistan is
faced with its worst crisis ever, not because political upheaval is unknown to this military-controlled nation, but
because it comes at a time of significant economic challenges.

On the afternoon of May 9, former Pakistan Prime Minister Imran Khan was arrested from the premises of the
Islamabad High Court by Pakistan Rangers, a paramilitary force. Later, a statement was issued that he was arrested
on a warrant issued by the National Accountability Bureau (NAB), for irregular acquisition of property which had been
deemed to fall under corrupt practices by a holder of public office, under Section 9 (a) of the NAB Ordinance, 1999,
leading to a loss of 190 million pounds to the state exchequer.

The accusation is of the sale of properties that are registered in the name of the Al Qadir Trust, of which both Imran
and his wife, Bushra Bibi, are the only trustees. The business tycoon involved had sold a property in Britain which,
ironically, he had purchased from Nawaz Sharif's son. The arrest was followed by massive violence across Pakistan,
with PTI workers and Imran Khan supporters taking to the streets and attacking army assets and properties. The
Pakistani judiciary came to Imran Khan's rescue not only by granting him bail in the Al Qadir Trust case but also
interim protection from any further action by the current dispensation.
Q 38. 30630558  What position did Shehbaz Sharif hold prior to becoming the Prime Minister of Pakistan?

a)  President of Pakistan b)  Chief Minister of Punjab

c)  Speaker of the National Assembly d)  Governor of Sindh

Q 39. 30630558  Which political party does Shehbaz Sharif belong to?

a)  Pakistan Tehreek-e-Insaf (PTI) b)  Pakistan Peoples Party (PPP)


c)  Pakistan Muslim League (N) (PML-N) d)  Pakistan Democratic Party

Q 40. 30630558  Who among the following served as the 22nd Prime Minister of Pakistan from August 2018 until April
2022?

a)  Shehbaz Sharif b)  Nawaz Sharif c)  Imran Khan d)  Asif Ali Zardari

Q 41. 30630558  Which political party did Imran Khan found and chair?

a)  Pakistan Muslim League (N) (PML-N) b)  Pakistan Peoples Party (PPP)

c)  Pakistan Tehreek-e-Insaf (PTI) d)  Muttahida Qaumi Movement (MQM)

Q 42. 30630558  In which sport did Imran Khan have a successful career before entering politics?

a)  Football b)  Hockey c)  Cricket d)  Tennis

Q 43. 30630558  What was the reason behind Imran Khan's removal from the office of Prime Minister?

a)  No-confidence motion b)  Corruption charges c)  Constitutional crisis d)  Assassination attempt

Q 44. 30630558  In which city did Imran Khan survive an assassination attempt?

a)  Lahore b)  Islamabad c)  Karachi d)  Wazirabad

Passage – 3

European Commission president [1] has said that the war in Ukraine has been a "stark reminder for Europe" that
"dependency on Russian fuel is not sustainable", emphasising on the transition toward home-grown renewable
energy.

In her speech at the International Solar Alliance in New Delhi, the EU chief said, "I am thinking about the war that
Russia has unleashed against Ukraine. For us, Europeans, it is a stark reminder that our dependency on Russian
fossil fuels is not sustainable… So, our transition to homegrown renewable energy is not only good for the
environment but also becomes a strategic investment in security. Energy policy is also security policy…"

Pointing out that "India and the European Union are on the same paths", the EU chief said that Prime Minister Modi
has already announced that India should be energy independent before it celebrates 100 years of its independence
as a country. "This would be in 2047. So it is in our common interest that this independence of fossil fuel that comes
from abroad is a transformation into renewable and clean energy," she said.

[1] added that India and the European Union are closely aligned in the fight against climate change and have
understood the importance of solar energy. "Both India and the European Union have embarked on their way to net-
zero. Europe wants to be climate neutral by 2050 and India by [2]. But in addition, India has committed to covering
half of its energy demand from renewable sources already by 2030. This is amazingly good news. This is very
ambitious," she said adding that Europe has set similar goals.
Q 45. 30630558  Who among the following has been serving as the 13th president of the European Commission
since 2019 whose name has been redacted with [1] in the passage above?

a)  Ursula von der Leyen b)  Angela Merkel c)  Thomas de Maizière d)  


Annegret Kramp-
Karrenbauer

Q 46. 30630558  How many member countries are there in the European Union (EU)?

a)  25 b)  26 c)  27 d)  29

Q 47. 30630558  Both India and the European Union have embarked on their way to net-zero. Europe wants to be
climate neutral by 2050 and India by [2]. Which of the following years has been redacted with [2] in the passage
above?

a)  2047 b)  2050 c)  2070 d)  2075

Q 48. 30630558  Which of the following countries is the latest country to leave the European Union?

a)  United Kingdom b)  France c)  Poland d)  Turkey

Q 49. 30630558  Which of the following statements is Not true regarding the European Union?

a)  
The eurozone is a group composed of the 21 EU member states that have fully implemented the economic and
monetary union and use the Euro as currency.
b)  Passport controls have been abolished for travel within the Schengen Area.
c)  The union and EU citizenship were established when the Maastricht Treaty came into force in 1993.

d)  The original member states of what came to be known as the European Communities were the Inner Six.

Q 50. 30630558  In 2022, which of the following countries is/are applied for European Union membership?

a)  Georgia b)  Moldova c)  Ukraine d)  All of the above

Q 51. 30630558  The latest country to join the Euro area (also referred to as 'Eurozone') was___________.

a)  Croatia b)  Poland c)  Latvia d)  Romania


Passage – 4

The tiger population in India grew by 200 from 2018 to 2022, according to the fifth cycle of the All India Tiger
Estimation (2022) released April 9, 2023. The number of tigers in India was [1] in 2022, up from 2,967 in 2018, the
report showed. The growth, however, slowed to 6.7 per cent in these four years from around 33 percent during 2014-
2018, analysis of the latest data showed. The report was released at an event organised to commemorate 50 years of
'Project Tiger' in Karnataka's Mysuru by Prime Minister Narendra Modi, who also launched the International Big Cats
Alliance the same day.

The tiger census covered forested habitats in 20 states of India. "A foot survey of 641,449 kilometres was done for
carnivore signs and prey abundance estimation. In these forests, 324,003 habitat plots were sampled for vegetation,
human impacts and ungulate dung," according to the authors of the report.

Camera traps were set up at 32,588 locations and generated 47,081,881 photographs, of which 97,399 were of
tigers, they noted. Parsed region-wise, the Shivalik Hills and Gangetic Plains saw a significant increase in the tiger
population, according to the latest census. As many as 804 individuals were photographed in the landscape spanning
several states that includes five important tiger reserves and several other protected areas. New areas in Uttar
Pradesh (Suhelwa Wildlife Sanctuary) and northwest of Yamuna in Himachal Pradesh have recorded photographic
evidence of tigers in the landscape, the report stated.
Q 52. 30630558  According to the latest census, what is the total tiger population in India in 2023 which has been
redacted with [1] in the passage above?

a)  2,967 b)  3,000 c)  3,167 d)  3,200

Q 53. 30630558  Which of the following is not one of the big cats protected under the International Big Cat Alliance?

a)  Tiger b)  Leopard c)  Lion d)  Giraffe

Q 54. 30630558  Which of the following states in India has the highest number of tigers?

a)  Madhya Pradesh b)  Tamil Nadu c)  Kerala d)  Rajasthan

Q 55. 30630558  Which of the following books was announced by Prime Minister Narendra Modi detailing the strategy
for tiger protection over the next 25 years?

a)  Tiger Vision b)  Tiger Resurgence


c)  Amrit Kaal Ka Tiger Vision d)  Project Tiger: A Vision for the Future

Q 56. 30630558  What is the primary method used in the tiger census to gather data?

a)  Satellite imaging b)  Ground-based surveys and camera traps


c)  Drone surveillance d)  DNA analysis

Q 57. 30630558  How often is the national tiger census conducted in India?

a)  Every year b)  Every three years c)  Every five years d)  Every four years

Q 58. 30630558  When was Project Tiger launched in India?

a)  1960 b)  1980 c)  1973 d)  1990

Passage – 5
Evading a direct reply on the possible inclusion of Ladakh under the Sixth Schedule of the Constitution, the Union
Home Ministry has informed a Parliamentary panel that the main objective of the inclusion of tribal populations under
the said schedule is to ensure their overall socio-economic development, which the Union Territory's administration
"has already been taking care of since its creation".

On August 5, 2019, the former State of Jammu & Kashmir was bifurcated into two Union Territories - Jammu &
Kashmir, and Ladakh, the latter without a Legislative Assembly. After its special status was removed, several political
groups in Ladakh have been demanding that land, employment, and the cultural identity of Ladakh, should be
protected under the Sixth Schedule. The Schedule protects tribal populations, providing autonomy to communities
through the creation of Autonomous Development Councils, which can frame laws on land, public health and
agriculture.

In 2021, Ladakh's only member in the Lok Sabha, the BJP's Jamyang Tsering Namgyal, demanded constitutional
safeguards by amending the Ladakh Autonomous Hill District Council (LAHDC) Act for the protection of land,
employment, and the cultural identity of Ladakh under the Sixth Schedule.

The Parliamentary Standing Committee on Home Affairs tabled a report in the Rajya Sabha which said that, according
to the 2011 Census, the tribal population in the Union Territory of Ladakh is 2,18,355, that is 79.61% of the total
population of 2,74,289. The committee recommended that special status may be granted to the Union Territory of
Ladakh considering the developmental requirements of the tribal population. The report said: "The Committee further
recommends that the possibility of including Ladakh in fifth or sixth Schedule may be examined."
Q 59. 30630558  The Sixth Schedule under Article 244 of the Constitution provides special provisions for the
administration of tribal areas in which of the following states?

a)  Assam, Meghalaya, Tripura, and Mizoram b)  Jammu and Kashmir and Ladakh
c)  Ladakh and Arunachal Pradesh d)  Ladakh, Assam, and Meghalaya

Q 60. 30630558  What does the Sixth Schedule provide in terms of legislative and administrative autonomy?

a)  Complete autonomy from the state and central governments


b)  Limited autonomy in legislative, judicial, and administrative matters within the state

c)  No autonomy, but certain modifications and exceptions in the application of laws

d)  Direct control and supervision by the President or Governor

Q 61. 30630558  What recommendation did the National Commission for Scheduled Tribes make regarding Ladakh?

a)  Inclusion in the Fifth Schedule b)  Inclusion in the Sixth Schedule

c)  Inclusion in the Special Category Status d)  Inclusion in the Scheduled Castes list

Q 62. 30630558  The Ministry of Home Affairs (MHA) constituted a high-powered committee for the Union Territory of
Ladakh to ensure:

a)  Protection of land and employment b)  Inclusion in the Sixth Schedule

c)  Revocation of Article 370 d)  Preservation of cultural heritage

Q 63. 30630558  The demand for inclusion in the Sixth Schedule by civil society groups in Ladakh stems from:

a)  Fear of big businesses and conglomerates taking away land and jobs

b)  The need for preservation of cultural heritage


c)  The recommendation of the National Commission for Scheduled Tribes

d)  The revocation of Article 370

Q 64. 30630558  What is the composition of the district council in each autonomous district under the Sixth
Schedule?

a)  20 members elected by the people and 10 members nominated by the Governor

b)  30 members, with four nominated by the Governor and 26 elected on the basis of adult franchise

c)  10 members elected by the people and 20 members nominated by the Governor
d)  26 members elected by the people and four members nominated by the Governor

Q 65. 30630558  The acts of Parliament or the state legislature do not apply to autonomous districts under the Sixth
Schedule, unless:

a)  The President or Governor directs otherwise b)  The district council approves them

c)  The Supreme Court intervenes d)  The local community requests their application

Legal Reasoning
Directions for questions 66 to 105: You have been given some passages followed by questions based on each
passage. You are required to choose the most appropriate option which follows from the passage. Only the
information given in the passage should be used for choosing the answer and no external knowledge of law
howsoever prominent is to be applied.

Disclaimer
All names and facts mentioned are hypothetical and are being used for educational purposes. Any resemblance with
any person, institution or situation is purely coincidental and unintentional.

Passage – 1

The principle of strict liability evolved in the case of Rylands v Fletcher. In the year 1868, the principle of strict liability
states that any person who keeps hazardous substances on his premises will be held responsible if such substances
escape the premises and causes any damage. Essentials of strict liability are:

• Dangerous Substances: The defendant will be held strictly liable only if a "dangerous" substance escapes from his
premises.
• Escape: One more essential condition to make the defendant strictly liable is that the material should escape from
the premises and should not be within the reach of the defendant after its escape.
• Non-natural Use: To constitute a strict liability, there should be a non-natural use of the land. Exception to the Rule of
Strict Liability

There are certain exceptions to the rule of strict liability, which are-
• Plaintiff's Fault: If the plaintiff is at fault and any damage is caused, the defendant would not be held liable, as the
plaintiff himself came in contact with the dangerous thing.
• Act of God: The phrase "act of God" can be defined as an event which is beyond the control of any human agency.
Such acts happen exclusively due to natural reasons and cannot be prevented even while exercising caution and
foresight.
• Act of the Third Party: The rule also doesn't apply when the damage is caused due to the act of a third party. The
third party means that the person is neither the servant of the defendant, nor the defendant has any contract with
them or control over their work. But where the acts of the third party can be foreseen, the defendant must take due
care. Otherwise, he will be held responsible.
• Consent of the Plaintiff: This exception follows the principle of volenti non fit injuria.
Q 66. 30630558  F lived adjacent to K. F runs a cold storage. L who also lived nearby, recently got a mosquito
repellent sprayed in the entire locality, as a precaution against the malarial outbreak. Due to the mosquito repellent
spray, the machine of the cold storage got damaged, and fruits inside the cold storage, began rotting with flies and
insects around it. K who lived nearby, too was annoyed by the smell, flies, and insects because of the rotting of fruits
in the close proximity. K filed a suit of strict liability against F, for running a cold storage nearby. Decide.

a)  This is not a case of strict liability, because it is an act of the third party.

b)  This is not a case of strict liability, because it is K's fault that he has not complained earlier.
c)  This is a case of strict liability, because running a cold storage in a residential area is unnatural use of land.

d)  This is a case of strict liability, because F was not diligent to prevent the damage of machines.

Q 67. 30630558  B had kept a furious dog as his pet, to warn him against any trespasser, in order to prevent high
rates of theft in the locality. One day, few children were playing in the park nearby and the ball fell on the dog, he
started barking. E, who threw the ball, went to the house to take it and was bitten by the dog. The parents of E, filed a
complaint against B for keeping an extraordinarily furious dog in their house. Decide.

a)  This is not a case of strict liability, because the ball hit the dog.

b)  This is not a case of strict liability, because the boy went inside the house.
c)  This is a case of strict liability, because the dog was extraordinarily furious.

d)  This is a case of strict liability, because there are other harmless methods to prevent trespassers.

Q 68. 30630558  Why is volenti non-fit injuria, treated as an exception to the rule of strict liability?

a)  It saves the erring party from the liability. b)  It desists the plaintiff to claim compensation.

c)  The party is assumed to be willing to take the injury. d)  The injury is caused due to the fault of the plaintiff.

Q 69. 30630558  X has a diamond of a rare quality. It is very expensive. In order to protect it he keeps in a special cell
with video cameras and armed security personnel. Also the entire cell is fenced with high voltage electric wires. A
passer-by, Kiran, tries to enter the cell. Not being aware that the fence is electrically charged, suffers a shock and
dies. A case under strict liability is brought against X. Decide.

a)  X is liable as there was no warning sign on the fence.

b)  X is not liable as diamond per se is not a dangerous thing.

c)  X is liable as he took too much security than was reasonably required.

d)  X is not liable as Kiran voluntarily subjected himself to the risk.

Q 70. 30630558  Geeta Chemicals Pvt. Ltd. was a registered company in Agra. It is engaged in the manufacture of
certain chemicals and fertilizers. Its uses various chemicals one of which is methyl isocyanine which is highly toxic for
human health and is stored in high quantities at the site. Because of an earthquake the container got dismantled and
the gas spreads to nearby areas killing anyone who comes in contact with it. Around 50,000 people died. It was later
found out that there was no negligence on part of the company employees. Is the company liable?

a)  Geeta Ltd. is liable for strict liability.


b)  Geeta Ltd. is liable for absolute liability.

c)  Geeta Ltd. is neither liable for strict liability nor for absolute liability.
d)  Geeta Ltd. is liable for negligence.

Directions for questions 66 to 105: You have been given some passages followed by questions based on each
passage. You are required to choose the most appropriate option which follows from the passage. Only the
information given in the passage should be used for choosing the answer and no external knowledge of law
howsoever prominent is to be applied.

Disclaimer
All names and facts mentioned are hypothetical and are being used for educational purposes. Any resemblance with
any person, institution or situation is purely coincidental and unintentional.

Passage – 2

Article 32 of the Indian Constitution gives the right to individuals to move to the Supreme Court to seek justice when
they feel that their right has been 'unduly deprived'. The apex court is given the authority to issue directions or orders
for the execution of any of the rights bestowed by the constitution as it is considered 'the protector and guarantor of
Fundamental Rights'. There are five types of Writs as provided under Article 32 of the Constitution, one of which is
writ of quo warranto:

Quo Warranto: Writ of Quo Warranto implies thereby "By what means". This writ is invoked in cases of public offices
and it is issued to restrain persons from acting in public office to which he is not entitled to. Although the term 'office'
here is different from 'seat' in legislature but still a writ of Quo Warranto can lie with respect to the post of Chief
Minister holding a office whereas a writ of quo warranto cannot be issued against a Chief Minister, if the petitioner
fails to show that the minister is not properly appointed or that he is not qualified by law to hold the office.

The Court has the discretion to refuse to grant quo warranto in cases where:
• The Court's interference would not change the end result;
• The case is vexatious;
• The defendant no longer wrongfully occupies public office.

The writ of quo warranto can be issued under the following cases:
• When a public office (created by law or the Constitution) is occupied by a private person, who does not actually have
the authority to do so.
• The public office must be substantive in character. The duties connected to the office must also be public in nature.
• The usurper, whose authority is being challenged, must be holding his position at the time the challenge is made.
• Even if a person was qualified at one point of time, the writ of quo warranto can be issued against him if he loses his
qualifications.
Q 71. 30630558  ABC, a college was owned by the richest businessmen in the country, who is a close ally of the
ruling party, had recently changed its vice chancellor. The new appointee is neutral to the political environment of the
college, which is detested by the ruling party. Within a month, he was removed and another person who is inclined to
the ruling party was inducted as the new vice chancellor. A writ of quo warranto was filed against the new
appointment. Decide.

a)  Writ of quo warranto would not hold because it is a private university.


b)  Writ of quo warranto would not hold because political influence is considered.
c)  Writ of quo warranto would hold because the university had a huge governmental influence.

d)  Writ of quo warranto would hold because the vice chancellor was removed within a month.

Q 72. 30630558  PYQ opened a vacancy for the head of human resource department in their company. They have
listed the necessary qualifications needed for the post, one of which was masters from a particular college, which was
best known for course in human resource. M, filed a writ of quo warranto by claiming that the company, is promoting a
particular college, which is one of the costliest fee structure, hence only meant for the elites, thus the company, is
exercising structural discrimination. Decide.

a)  The writ would not stand because it does not concern with discrimination.

b)  The writ would not stand because an institution could lay out necessary qualification for its post.

c)  The writ would stand because it is an example of abject discrimination.


d)  The writ would stand because it is promoting a particular college.

Q 73. 30630558  H, was a teacher in a medical college, who was appointed by fraud. J, a student was taught by H,
had performed many failed surgeries and operations, due to which, his medical practise never succeeded. J's parent
filed a suit of quo warranto against the teacher, mentioning that because of him, their child could not succeed in
medical profession. Decide.

a)  The writ would fail because the parents have no locus standi to file the case.

b)  The writ would fail because H was removed from the post.
c)  The writ would stand, because parents have the locus standi to file a case.

d)  The writ would stand, because J's life has been affected due to H.

Q 74. 30630558  On which grounds, could the writ of quo warranto be rejected?

a)  The interference of the judiciary might have effect on the end result.

b)  The defendant is still an occupant of the position that he has been wrongfully occupying.

c)  The case would not yield any result.


d)  The case would be vexatious.

Q 75. 30630558  In light of increasing menace a law was passed by the State government that for filing a writ petition
for enforcement of fundamental rights a mandatory fee of Rs. 50,000 will be paid by the petitioners. This raised a lot
of hue and cry and one poor fellow challenged the arbitrary decision of the State. Decide.

a)  
He shall fail because the State has not prohibited anyone from filing petitions before the Supreme Court, they have
only provided for court fee to be paid as money is required to run the courts too.
b)  He shall fail because the restriction is neither unreasonable nor restrictive.

c)  
He shall succeed because this law effectively prohibits the citizens who cannot afford to pay heavy court fee from
approaching the Supreme Court and thereby restricting the exercise of the right to approach the Supreme Court.
d)  
He shall succeed because no court fee can be imposed writ petitions as it is a fundamental right to file writ petitions
before the Supreme Court.

Directions for questions 66 to 105: You have been given some passages followed by questions based on each
passage. You are required to choose the most appropriate option which follows from the passage. Only the
information given in the passage should be used for choosing the answer and no external knowledge of law
howsoever prominent is to be applied.

Disclaimer
All names and facts mentioned are hypothetical and are being used for educational purposes. Any resemblance with
any person, institution or situation is purely coincidental and unintentional.

Passage – 3

Kidnapping means taking away a person against his/her will by force, threat or deceit. Usually, the purpose of
kidnapping is to get a ransom, or for some political or other purposes etc. According to section 360, if any person
takes a person beyond the limits of India against the consent of that person or against the consent of someone who is
legally entitled to give consent on that person's behalf, then the offence of kidnapping from India is committed.
Section 361 explains kidnapping from lawful guardianship. According to this section, if a person takes away or entices
a minor (i.e, a boy under the age of 16 years and a girl under the age of 18 years) or a person of unsound mind, away
from his/her lawful guardian without the guardian's consent, then that person commits the offence of kidnapping from
lawful guardianship. It says that it does not result in the crime of kidnapping from lawful guardianship, if the person in
good faith, i.e, honestly with reason, believes that:

• He is entitled to the lawful custody of the child; or


• He is the father of an illegitimate child.

Section 363 of the Indian Penal Code lays down the punishment for both kinds of kidnapping (Kidnapping from India
and Kidnapping from lawful guardianship).

• The punishment prescribed in this section is :


• Imprisonment of either description which can extend up to seven years, and Fine.

Imprisonment of either term means either of the two imprisonments prescribed in the Indian Penal Code:
• Simple Imprisonment: This means that during the imprisonment, the prisoner is idle and is not required to do any
hard labour.
• Rigorous Imprisonment: This means that during the imprisonment, the prisoner must engage in hard labour.
Q 76. 30630558  N and M were friends for a long time. M has been persisting N to get married to him, however N
would reject the proposal because she was 17 years old, a minor. However one day, N could not negate M's
persistent requests of marriage and ran away from her family and in order to marry M. N's family sued M for
kidnapping. Decide.

a)  M is not liable for kidnapping, because N ran away from her house on her own.

b)  M is not liable for kidnapping because N knew about the consequence of her act.
c)  M is liable for kidnapping because N was tired to M's persistent requests.

d)  M is liable for kidnapping because N had rejected the proposal multiple times in the past.

Q 77. 30630558  J and H were childhood friends both 17 years of age. J told H that he had opened a new company,
and if H agrees to become a receptionist in his company, he could give her handsome salary, and many other perks.
H was very excited and readily agreed to the proposal, however her family was suspicious and refused to give her the
permission to join J's company. H, however sneaked out of the house at late night. When H went to the location, she
found that J owned no company, rather he dealt in human trafficking. H's family filed a case of kidnapping against J.
Decide.

a)  J is liable for kidnapping, because he enticed H for running away from her family.

b)  J is liable for kidnapping, because he lied about the company to H.

c)  J is not liable for kidnapping, because he did not coerced H to leave the family.

d)  J is not liable for kidnapping, because H left the house despite her parents' objections.

Q 78. 30630558  D was recently told by his great grandfather that he had a son, who was lost in a fete. The son has
the same crescent shaped body mark as that of D. One day, while roaming on the street, he saw a child that had the
same shaped birthmark, since the shape of the birth mark is unusual and not very common, therefore D assumed that
it is his child, and took the child with him, on the pretext of giving him a chocolate.

The son's guardian filed a suit against D for kidnapping. D claimed that it is his child. Decide.

a)  D is liable for kidnapping, because he took the child away from the custody of his guardian.

b)  D is liable for kidnapping, because he enticed him on the pretext of giving him chocolates.

c)  D is not liable for kidnapping because he thought that he was his son.
d)  D is not liable for kidnapping, because he is the real father.

Q 79. 30630558  Which of the following is true regarding kidnapping out of India?

a)  The victim has given consent to be taken out of India.

b)  The victim is a minor.

c)  The victim has not consented to be taken away, but his guardians have given the permission to do so.
d)  The victim could be of any nationality.

Q 80. 30630558  Which is not an essential of taking a minor away from a legal guardianship?

a)  Taking or enticing a person of unsound mind or a minor.


b)  Taking away from a lawful guardian.

c)  The legal guardian has not permitted the taking away of the victim.

d)  The victim has not consented, however the guardian has permitted the victim to be taken away.

Directions for questions 66 to 105: You have been given some passages followed by questions based on each
passage. You are required to choose the most appropriate option which follows from the passage. Only the
information given in the passage should be used for choosing the answer and no external knowledge of law
howsoever prominent is to be applied.

Disclaimer
All names and facts mentioned are hypothetical and are being used for educational purposes. Any resemblance with
any person, institution or situation is purely coincidental and unintentional.

Passage – 4

"When, at the desire of the promisor, the promise or any other person has done or abstained from doing, or does or
abstains from doing, or promises to do or to abstain from doing something, such act or abstention or promise is called
a consideration for the promise."

For the valid contract, the consideration and the object should be lawful. Every agreement where the object and
consideration is unlawful then the agreement is void. As per Section 23 of the Indian Contract Act, 1872 What
considerations and objects are lawful and what not:

Cases where the consideration or object considered to be unlawful:


• It is forbidden by law: When something is forbidden by law, an agreement to do that is unlawful. An agreement to
do what has been forbidden by the Indian Penal Code or by some other law cannot be enforced. A contract to pay
some money if a crime or tort is committed is not enforceable. If the contract specifically indemnifying a person
against liability for an intentional wrong like deceit, it is unlawful. An agreement offending a public policy is void from
the starting and the same cannot become valid even if the parties agree to that effect.
• It would defeat the provisions of any law: If the object or consideration of an agreement is of such a nature that, if
it is permitted, it would defeat the provisions of any law, such an agreement is void. Certain acts may not be expressly
forbidden by law, but if they result in circumventing any law, they cannot be encouraged.
• It is fraudulent purpose: If the consideration or object of an agreement is to commit fraud, the agreement is void.
• It involves or implies injury to the person or property of another: If the consideration or the object of an
agreement is to cause an injury to the person or property of another person, then the agreement is unlawful and
hence it is void. Injury here means unlawful harm, for example, an agreement to commit fraud or a tort.
• Immoral: If the consideration or object an agreement is regarded by the court to be immoral or opposed to public
policy, the agreement is unlawful and the same has also been declared void.
Q 81. 30630558  G and H entered into a contract, wherein, G would give him a loan, if H would divorce her husband
and marry him. Decide the validity of the contract.

a)  The contract is invalid because it is against the public morality.


b)  The contract is invalid because G is luring H for money.

c)  The contract is valid because G would marry H only after getting him divorced.

d)  The contract is valid because it is against adulterous behaviour.

Q 82. 30630558  A approached T, an IT firm, wherein he wanted to make a deal to sell semiconductor chips, however
the officials at T, came to realise that A's reputation in the market is not good, and hence refused to enter into a
contract with him. However later A sent B to guise itself as the representative of another company, and present the
offer to T. The offer was accepted by T, in the guise of B's company, A would honour the contract. On the basis of the
above facts, decide the validity of the contract.

a)  The contract is invalid because B lied to T.

b)  The contract is invalid because A has committed fraud.


c)  The contract is valid, because the semiconductor chips are of good quality.

d)  The contract is valid, because T agreed to the terms of the contract.

Q 83. 30630558  Partners of a firm, contracted an accountant to help the firm avoid taxes. However, at the end of the
financial year, the firm had to pay more taxes than the previous year. The partners, held the accountant liable for
breaking the contract, which was to help them avoid taxes. The accountant however claimed that tax avoidance is
illegal and thus the contract is invalid. Decide.

a)  The contract is invalid because it was based on illegal consideration of tax avoidance.

b)  The contract is invalid because the accountant is not competent enough.


c)  The contract is valid because tax avoidance is legal.

d)  The contract is valid because tax avoidance cannot be predicted.

Q 84. 30630558  Tina told her maid to arrive an hour early in the morning each day as she was getting late for office
and spoiling her impression in front of her boss. Her maid agreed to do this from the next morning. However, she got
stuck in traffic and couldn’t arrive early and Tina had to wait due to which she became extremely late for office. This
put a wrong impression on her boss and she was finally removed from office. Tina sued her maid for not being on
time. Decide.

a)  
There was an offer from Tina which was accepted by the maid. Necessary terms of agreement have been fulfilled and
an agreement has come into place.
b)  As a consideration, arriving early has no value in the eyes of law. Hence, the agreement is void.

c)  Tina may sue her maid but will not be granted any relief as no penalty is awarded in such matters.
d)  There is no consideration in the given case. Due to absence of consideration, no agreement made.

Q 85. 30630558  A was a worker on contract basis in B's factory. On several counts of negligence in performance of
his professional duties, B decided not to renew the contract with A when it duly expired. Coming to know of this, C, a
good friend of A and also a worker in B's factory went to B and promised him that he will do all the work that A lacked.
On this promise of C's, B renewed the contract for services with A. However, later, C refuses to compensate for the
work that's not done by A. B claims that C's promise is his consideration for the contract and claims performance of
the same. Is C liable?

a)  No, as there is no contract between B and C. b)  Yes, as B renewed A's contract based on C's promise
c)   d)  Both (a) and (c)
No, as C has not made the promise at the A's instance.

Directions for questions 66 to 105: You have been given some passages followed by questions based on each
passage. You are required to choose the most appropriate option which follows from the passage. Only the
information given in the passage should be used for choosing the answer and no external knowledge of law
howsoever prominent is to be applied.

Disclaimer
All names and facts mentioned are hypothetical and are being used for educational purposes. Any resemblance with
any person, institution or situation is purely coincidental and unintentional.

Passage – 5

Every person has rights and obligations that he or she enjoys while living in a state. A doctrine is a principle, set of
rules, or position usually used and upheld by the courts of law. The doctrine of laches is a legal maxim that supports
the person who is aware of their legal right, pointing to denying access to those who are careless in filing a suit in due
time. It will not provide any help to the applicants who delayed their filing unreasonably. The doctrine of laches is used
by the courts to deal with an inordinate delay that is occurring in filing a petition or complaint. It means if you have any
legal claim, you have to approach the court promptly. Laches is a fair doctrine or an equitable defense. The courts will
not help the person who sleeps over their rights but help those who are aware of their rights. A person is said to be
liable for laches when he comes to the court to affirm their rights after a reasonable delay in that respect. The person
who is asserting laches has the burden of proving that it is applicable. In many matters, a delay in filing a case has
the effect of blocking the opposing party from putting on a fair defense.
Because of that delay, evidence disappears, memories scruple, and witnesses go their ways. The person who wants
to claim the doctrine or think of it as help opens the door of hope, but in legal terms, the word laches is an opportunity
that is now lost. Some elements must be satisfied to consider this doctrine to bar the petitioner from the cause of
action:

• Delay must be unreasonable at the time of bringing the matter;


• Negligence in asserting a claim or right;
• Knowledge of a claim by the petitioners in advance

When a person comes across the doctrine of laches, a sudden question pops up in mind that the statute of limitation
and doctrine of laches are alike. The genesis of both statutes is to ensure that the claims should be brought within a
reasonable period so that witnesses and evidence can be found easily. Statute of limitation is a law that provides
maximum time to the parties who are involved in a dispute to initiate legal proceedings from the date of an offense
committed. The purpose of laches is to ensure that the claims are brought in a reasonable or appropriate time as that
will also help to contact witnesses and find evidence easily.
Q 86. 30630558  Z started with a construction at a land, that has been owned by Y. Despite knowing this, Y sued Z
after 18 months. She defended herself, by claiming that she has been caught up by some personal issues. Decide.

a)  Y is barred by laches act, because she has taken a long time to sue Z.
b)  Y is barred by laches act, because there are no exceptions, under this statute.

c)  Y is not barred by laches act, because she has been caught by some personal issues.

d)  Y is not barred by laches act, because she had the knowledge about the construction work.

Q 87. 30630558  F got hold of a casket full of gold, worth billions of dollars, at H's home. H knew about this theft and
did not sue F, until after 3 years. When enquired about the delay, H explained that he waited to see, the expenditure
of that gold. Decide.

a)  F is barred by laches because he stored, casket of gold at his house.

b)  F is barred by laches because he knew about the theft for 3 years.
c)  F is not barred by laches because he waited to see the use of the gold in the casket.

d)  F is not barred by laches because spending billions of worth takes time.

Q 88. 30630558  Which of the following element is not necessarily to be satisfied for the application of doctrine of
laches?

a)  Negligence is claiming the criminal act. b)  Delay in filing a suit.


c)  Inequitable defence for the erring party. d)  Knowledge of the claim by the petitioners.

Q 89. 30630558  What is the difference between laches and statute of limitation?

a)  
Doctrine of laches is with the perspective of the petitioner, and the statute of limitation is with regards to the
defendant.
b)  Doctrine of laches is used in criminal cases, and the statute of limitation is with respect to civil cases.

c)  Doctrine of laches is ending the dispute, whereas the statue of limitation is to initiate a legal proceeding.

d)  Doctrine of laches is to buy time for investigation, whereas the statute of limitation is to initiate a legal proceeding.
Q 90. 30630558  L filed a case of trademark infringement against K's company. However K claimed that his
company's trademark has been in the market for years, and L being waken up to this fact now, bars her from getting
an injunction against his company, because of the doctrine of laches. Decide.

a)  L would get an injunction, because public interest is more important that individual's right.
b)  L would not get an injunction, because intellectual property rights, are outside the scope of doctrine of laches.

c)  L would get an injunction, because an offender must be punished.

d)  L would not get an injunction, because L is assumed to have known about the existence of the fake trademark.

Directions for questions 66 to 105: You have been given some passages followed by questions based on each
passage. You are required to choose the most appropriate option which follows from the passage. Only the
information given in the passage should be used for choosing the answer and no external knowledge of law
howsoever prominent is to be applied.

Disclaimer
All names and facts mentioned are hypothetical and are being used for educational purposes. Any resemblance with
any person, institution or situation is purely coincidental and unintentional.

Passage – 6

An assault is a threat or attempt to do a corporeal hurt to another. Three elements must be established in order to
establish tortious assault: first, the plaintiff apprehended immediate physical contact, second, the plaintiff had
reasonable apprehension (the requisite state of mind) and third, the defendant's act of interference was intentional
(the defendant intended the resulting apprehension). But intent for purposes of civil assault can be either general or
specific. Specific intent means that when the defendant acted, he or she intended to cause apprehension of a harmful
or unwanted contact. General intent means that the defendant knew with substantial certainty that the action would
put someone in apprehension of a harmful or unwanted contact. Battery is the intentional and direct application of any
physical force to the person of another. It is the actual striking of another person, or touching him in a rude, angry,
revengeful, or insolent manner. Battery requires actual contact with the body of another person so a seizing and
laying hold of a person so as to restrain him; spitting in the face; throwing over a chair; or carriage in which another
person is sitting; throwing water over a person; striking a horse so that it bolts and throws its rider; taking a person by
the collar; causing another to be medically examined against his/her will; are all held to amount to battery. The fact
that the wrongdoer has been fined by a criminal court for the assault is no bar to a civil action against him for
damages. The previous conviction of the wrongdoer in a criminal court is no evidence of assault. The factum of the
assault must be tried in a civil court, which is not bound by conviction or acquittal in criminal proceedings. A plea of
guilty in a criminal court may, but a verdict of conviction cannot, be considered in evidence in a civil court.
Q 91. 30630558  A five-year-old boy, Dailey intentionally to hurt pulled a chair out from under Garratt just as she was
about to sit causing her to fall and break her hip. Garratt brought suit for personal injuries and alleged that Dailey had
acted deliberately. Decide-

a)  Dailey had assaulted Garratt and thus is liable.

b)  Dailey is not liable as she is a minor.


c)  
Dailey had committed battery by pulling chair of Garratt and it would be immaterial whether she intended to hurt her.
d)  
Dailey had committed battery as the element of intent is satisfied if the defendant knows with a substantial certainty
that his act will result in a harmful or offensive contact.
Q 92. 30630558  Talmage and several other children were playing on the roofs of sheds on Smith's property. Smith
ordered the children to get down and threw a stick at Aman. The stick missed its intended target and struck Talmage
in the eye. Talmage lost all sight in the eye and sued for battery to recover for personal injuries. Decide-

a)  Smith is liable as he intended to inflict battery on a party.


b)  Smith is not liable as he did not intend to inflict injury to Talmage but to Aman.

c)  Smith is liable to Aman and not Talmage.

d)  Smith is not liable as he intended to frighten children (assault) and not hit them.

Q 93. 30630558  Deep and Anu had a verbal fight during meeting. Deep started abusing Anu which ignited him and
he clinched his fist towards Deep but was stopped by Kim. Decide whether Anu is liable for battery?

a)  Anu is liable for assault.

b)  
Anu is liable as he was advancing with an intention to hit and if it was not for Kim, Deep would have been injured by
now.

c)  Anu is not liable as it was Deep who abused Anu first which ignited him.

d)  Anu is not liable for battery.

Q 94. 30630558  Dada and Pada were playing chess. A fight broke between both and Dada frighten to Pada saying
that he will cut his throat and proceeds towards picking a knife. Decide-

a)  It is tort of battery on side of Dada as he picked the knife.

b)  It is tort of assault on side of Dada as he picked the knife.

c)  It is tort of battery on side of Dada as he frightened Pada to cut his throat.

d)  It is tort of assault on side of Dada as he frightened Pada to cut his throat.

Q 95. 30630558  Cullison met 16-year-old Sandy Medley in a grocery store parking lot. He invited her to have a soda
with him and to come to his home for a conversation. A few hours later he was awoken by a knock at his door. He
was confronted by Sandy Medley, her father Ernest, her brother, brother-in-law, and mother. Ernest had a revolver in
a holster strapped to his thigh. Sandy called him a pervert and her mother berated him. Ernest kept grabbing and
shaking the gun while still in the holster and threatening to jump astraddle of him if he did not leave Sandy alone.
Although no one touched Cullison, he feared he was about to be shot because Ernest kept grabbing the gun as if to
draw it from the holster while threatening him. As a result of this incident, Cullison sought psychological help to deal
with nervousness, depression and sleeplessness He sued the Medley family for battery.

a)  Cullison will get damages as Medleys assaulted him.


b)  Cullison will not get damages.

c)  
Cullison will get damages as he suffered nervousness, depression, sleeplessness, inability to concentrate, and
impotency.
d)  Cullison will not get damages as it was not assault.

Directions for questions 66 to 105: You have been given some passages followed by questions based on each
passage. You are required to choose the most appropriate option which follows from the passage. Only the
information given in the passage should be used for choosing the answer and no external knowledge of law
howsoever prominent is to be applied.
Disclaimer
All names and facts mentioned are hypothetical and are being used for educational purposes. Any resemblance with
any person, institution or situation is purely coincidental and unintentional.

Passage – 7

According to Winfield and Jolowicz, Negligence is the breach of a legal duty of care by the plaintiff which results in
undesired damage to the plaintiff.

In Blyth v. Birmingham Water Works Co, Negligence was defined as the omission to do something which a
reasonable man would do or doing something which a prudent or reasonable man would not do.

It can be characterized in three forms-


Nonfeasance: It means the act of failure to do something which a person should have done.
Misfeasance: It means the act of not doing an action properly when it should have been done properly.
Malfeasance: It means the act of doing something which should not have been done in the first place itself.

To commit the tort of negligence, there are primarily main essentials that are required. An act will be categorized as
negligence only if, all the conditions are satisfied namely -
(1) Duty Of Care: It is one of the essential conditions of negligence in order to make the person liable. It means that
every person owes, a duty of care, to another person while performing an act. Although this duty exists in all acts, but
in negligence, the duty is legal in nature and cannot be illegal or unlawful and also cannot be of moral, ethical or
religious nature.
(2) The Duty must be towards the plaintiff: A duty arises when the law recognizes a relationship between the
defendant and the plaintiff and requires the defendant to act in a certain manner toward the plaintiff.
(3) Breach of Duty to take care: It's not enough for a plaintiff to prove that the defendant owed him a duty of care but
he must also establish that the defendant breached his duty to the plaintiff. A defendant breaches such a duty by
failing to exercise reasonable care in fulfilling the duty.
Q 96. 30630558  Zoya, an owner of a big dog requests her friend Xiyu to take care of the dog while she is away due
to some urgent business meeting. Xiyu goes for shopping to new mall as there was sale which was only for 3 hours,
leaving the dog unattended who attacks a passer-by Bobby badly injuring him. Decide-

a)  Zoya is liable for tort of negligence as being owner of dog she cannot leave it under someone else.

b)  Bobby is liable for tort of negligence as he passed the road negligently.

c)  Xiya is liable, as dog being under her possession during that period, she owned a duty of care towards others.
d)  Xiyu is not liable as sale was only for 3 hours thus it was an emergency to leave dog.

Q 97. 30630558  Sham was getting married to Jiya. On date of a marriage a decorator was engaged to carry out
decorations in a house. Soon after, the decorator left the house without locking the doors or informing anyone. During
his absence, a thief entered the house and stole some property the value of which Sham claimed from the decorator.
Decide-

a)  Decorator is not liable as it was house of Sham thus he owned duty of care and not decorator.

b)  Decorator was liable as he was negligent in leaving the house open and failed his duty of care.

c)  Decorator should have informed Sham before leaving the house.


d)  
During function, a family member should look after all the workers and not let anyone work on their own, thus the
decorators are not liable.

Q 98. 30630558  X, a pregnant woman got down from a tram car. A motor-cyclist who was passing the tram collided
with a motor car which was on the other side of the tram. The motorcyclist died instantly and X could not witness the
accident. She had only heard the sound of the collision and once the body had been removed from the place of
accident, she visited the place and saw some blood which was left on the road. As a reaction to this incident, she
suffered a nervous shock and gave birth to a still-born child of 8 months because of which she sued the driver of the
car. Decide-

a)  The driver is liable as he owed a duty to care towards X.

b)  The driver is not liable as he did not own duty of care towards X.

c)  Nervous shock was result of X's own actions as no one asked her to visit place of accident.
d)  The driver is not liable as such consequences were not foreseeable.

Q 99. 30630558  Haya and Sawan are childhood friends. Sawan moved to London and after a gap of 3 years is re-
visiting India to meet Haya. Haya in order to make this meeting special buys a packed box of sweets from a well-
known shop. When Sawan visits Haya she offers her the sweet box. Sawan consumed the sweets but it was later
found that there were dead ants inside the box. Sawan due to consuming those sweets felt sick and brought an action
against sweet shop. Decide-

a)  Sweet shop sold box to Haya thus owed duty of care towards her and not Sawan.

b)  Sawan cannot sue the sweetshop as there was no breach of duty of their part.
c)  Sawan is not an Indian citizen now so he cannot sue the sweetshop.

d)  Sawan can sue sweet shop and is entitled for damages.

Q 100. 30630558  The post office of Malgudi Nagar was quite old. Though they were good with delivering letters they
did not care to maintain premises of post office. One day when Shamavati was passing through the premises, the
compound wall of post office collapsed and caused many injuries to Shamavati Decide-

a)  The post authorities are liable that had a duty to maintain the post office premises.

b)  The post authorities are not liable as their duty is only towards letters and not building.

c)  Shamavati should have been careful while passing post office.


d)  None of the above.

Directions for questions 66 to 105: You have been given some passages followed by questions based on each
passage. You are required to choose the most appropriate option which follows from the passage. Only the
information given in the passage should be used for choosing the answer and no external knowledge of law
howsoever prominent is to be applied.

Disclaimer
All names and facts mentioned are hypothetical and are being used for educational purposes. Any resemblance with
any person, institution or situation is purely coincidental and unintentional.

Passage – 8

A person in possession of a property is entitled to its undisturbed enjoyment as per law. However, if someone else's
improper use or enjoyment in his property ends up resulting into an unlawful interference with his enjoyment or use of
that property or of some of the rights over it, or in connection with it, we can say that the tort of nuisance has
occurred.

The word "nuisance" has been derived from the Old French word "nuire" which means "to cause harm, or to hurt, or
to annoy". The Latin word for nuisance is "nocere" which means "to cause harm".

Nuisance is an injury to the right of a person's possession of his property to undisturbed enjoyment of it and results
from an improper usage by another individual.

1. Public Nuisance: The Indian Penal code defines nuisance as an act which causes any common injury, danger or
annoyance, to the people in general who dwell or occupy the property, in the vicinity, or which must necessarily cause
injury, obstruction, danger, or annoyance to the people who may have occasion to use any public right.
2. Private Nuisance: Private Nuisance is that kind of nuisance in which a person's use or enjoyment of his property is
ruined by another. It may also injuriously affect the owner of the property by physically injuring his property or by
affecting the enjoyment of the property.

There are three kinds of remedies available in the case of a nuisance, these are: Injunction, damages, abatement.
Q 101. 30630558  A had wrongfully obstructed a public navigable creek which obstructed B from transporting his
goods through the creek due to which he had to transport his good through land and he suffered extra costs in the
transportation. Decide-

a)  A has caused public nuisance as a result of his act. b)  A has caused private nuisance as a result of his act.

c)  A has caused trespass as a result of his act. d)  A has caused negligence as a result of his act.

Q 102. 30630558  Mr. Gur Prasad Saxena filed a suit against Mr Radhey Shyam and five other individuals for
permanent injunction restraining them from installing and running a flour mill in the premises occupied by them. Gur
Prasad Saxena filed another suit against Radhey Shyam and five other individuals for a permanent injunction from
running and continuing to run an oil expeller plant. Mr. Gur Prasad Saxena has alleged that the mill was causing a lot
of noise which in turn was affecting his health.

a)  This cannot be covered under nuisance as it doesn't fulfil its essentials.


b)  This can be covered under nuisance.

c)  This can be covered under tort of trespass.

d)  This cannot be covered under tort of trespass.

Q 103. 30630558  Ace and Beck are neighbors and constantly fights over some issue or other. Beck has a poisonous
tree on his land which overtime outgrows and reaches the land of Ace. Now Ace is getting angry over it and warns
Beck to cut it but he declines. Ace files a civil suit for nuisance. Decide-

a)  Ace has no legal remedy available as being physical, it is trespass.

b)  Ace can go to Beck's land without his permission, and chops off the entire tree.

c)  Ace need to prove special damage as it is public nuisance.

d)  Ace has every right to cut that part of the tree which is affect his enjoyment of his land with prior notice to Beck.

Q 104. 30630558  Prem had sued the director for nuisance and asked permanent injunction to restrain the director
from showing a movie named "Good God". It was said by Prem that the contents of the movie significantly hurt the
religious sentiments of the people belonging to the Nota community as well as the religious sentiments of the Prem as
the movie showed Nota Goddess' to be jealous of one another and were ridiculed in the film.

a)  It is case of private nuisance as no one other than Prem complained.

b)  It is case of public nuisance as movie was released publicly.


c)  Permanent injunction cannot be granted in this case.

d)  Permanent injunction can be granted in this case.

Q 105. 30630558  What is difference between trespass and nuisance?

a)  
Nuisance, on one hand, is the direct physical interference with the plaintiff's possession of the property through some
material or tangible object whereas, in the case of a trespass, it is an injury to some right of the possession of the
property but not the possession itself
b)  
Trespass, on one hand, is the direct physical interference with the plaintiff's possession of the property through some
material or tangible object whereas, in the case of a nuisance, it is an injury to some right of the possession of the
property but not the possession itself.
c)  
Trespass is not actionable per se (actions which do not require allegations or proof), whereas, in the case of a
nuisance, only the proof of actual damage to the property is required.
d)  Both (b) and (c).

Logical Reasoning
Direction for questions 106 to 135: Read the following passages and answer the questions that follow.

Passage – 1

For 30 years, Omar al-Bashir, a former military officer, ruled Sudan with an iron hand and indiscriminate violence.
When he was toppled in April 2019 in a mass uprising, many hoped that the resource-rich country in the Horn of
Africa would finally get a chance to move towards a freer society with a representative and responsive administration.
But the tragedy of Sudan is that the monstrous regime that Mr. Bashir built outlasted his reign. Within two years of his
fall, the military was back, and now, a power struggle between the top two generals has pushed Sudan to the brink of
a civil war. Dozens of civilians have already been killed in fighting that broke out on Saturday in Khartoum and other
parts of the country between the military and the Rapid Support Forces (RSF), a notorious paramilitary group. Despite
international calls for truce, Lt.Gen. Abdel Fattah al-Burhan, the military chief as well as the head of the Sovereignty
Council, the transitional administration, and his deputy, Lt.Gen. Mohamed Hamdan Dagalo, who commands the RSF,
have refused to negotiate, blaming each other for the attacks. Mr. Dagalo, who has close ties with Russia's Wagner
private military company and Saudi Arabia, claims that the RSF has taken control of the presidential palace and has
vowed to bring Gen. Burhan to justice, while the military has dismissed such claims and launched air strikes against
RSF sites.

Just two years ago, the two generals stood hand in hand when they ousted a civilian transition government and took
over the reins of the country. Faced with international isolation and domestic pressure, they agreed to transfer power
back to the civilians. But differences emerged on who should control the post-transition military. Gen. Burhan supports
the integration of the RSF into the regular military and transition to civilian government to take place in two years,
while Gen. Dagalo, who fears that he would lose his clout, wants to delay it by 10 years. Discord grew into mistrust
and mistrust led to fighting. And the fighting could drag the country, which has a history of internal strife, into an all-out
civil war. Sudan's generals are known for their scant regard for the welfare of their people. The country is struggling
with an economic crisis, with rocketing inflation and a burning hunger problem. The last thing Sudan wants now is a
civil war. If the priority of the generals is to address Sudan's basic problems, they should pay attention to the call for a
truce and dialogue, and commit themselves to a timeline-sensitive democratic transition. Decades of military rule in
Sudan have resulted in a lot of atrocities. Generals Burhan and Dagalo should not tread the same course.
Q 106. 30630558  What is the primary cause of the power struggle between Lt. Gen. Abdel Fattah al-Burhan and Lt.
Gen. Mohamed Hamdan Dagalo that has led to fighting and threatens to plunge Sudan into civil war?

a)  
Differences in the approach of the two generals to solving the economic crisis, with Gen. Burhan advocating for
international aid and Gen. Dagalo promoting self-sufficiency.
b)  
Differing views of the two generals on the timeline for the transition to civilian government and the integration of the
RSF into the regular military.
c)  Conflicting loyalties between the two generals supporting separate entities.
d)  
Gen. Burhan's insistence on establishing a democratic government immediately, while Gen. Dagalo seeks to maintain
military control and suppress civilian participation in politics.

Q 107. 30630558  As an international mediator seeking to prevent a civil war in Sudan, which of the following
strategies would be most directly aligned with the underlying issues causing the power struggle between the two
generals as described in the passage?

a)  
Offering economic incentives to the Sudanese government to prioritize addressing the nation's economic crisis and
hunger problem.
b)  Facilitating dialogue between the generals to negotiate a mutually agreeable timeline.

c)  Encouraging the United Nations to impose sanctions on both generals to pressure them into resuming democracy.

d)  
Organizing a peace conference between the generals and various Sudanese political factions to address broader
concerns about governance and military control in the post-transition period.

Q 108. 30630558  Which of the following, if true, would most weaken the author's arguments regarding the power
struggle between Lt. Gen. Abdel Fattah al-Burhan and Lt. Gen. Mohamed Hamdan Dagalo?

a)  
The underlying cause of the power struggle is not the timeline for civilian transition and RSF integration, but personal
ambitions and desire for wealth.
b)  
The Rapid Support Forces (RSF) have a history of committing human rights abuses under the command of Gen.
Dagalo, leading to increased tensions.
c)  
Gen. Burhan and Gen. Dagalo have previously cooperated on military operations, indicating a potential for future
collaboration despite their disagreements.
d)  
The international community has shown a strong willingness to intervene and negotiate a truce between the
conflicting parties in Sudan.

Q 109. 30630558  Which of the following, if true, would most strengthen the author's arguments regarding the power
struggle between Lt. Gen. Abdel Fattah al-Burhan and Lt. Gen. Mohamed Hamdan Dagalo?

a)  
Both generals have been secretly negotiating with various international stakeholders to gain support for their
respective positions on the timeline.
b)  
Gen. Burhan and Gen. Dagalo have had a long-standing rivalry in the Sudanese military, which has been exacerbated
by the current political situation.
c)  
The Sudanese public overwhelmingly supports one general over the other, leading to increased tensions and division
among the population.
d)  
Other Sudanese military factions are starting to align themselves with either Gen. Burhan or Gen. Dagalo based on
their stances on the timeline.

Q 110. 30630558  Which of the following is the author most likely to disagree with?

a)  
The power struggle between Gen. Burhan and Gen. Dagalo is primarily driven by their personal ambitions rather than
disagreements on the timeline for civilian transition and RSF integration.
b)  The potential for an all-out civil war in Sudan is a real concern due to the country's history of internal strife.

c)  
A truce and dialogue are necessary to prevent further violence and commit the generals to a timeline-sensitive
democratic transition.
d)  
Decades of military rule in Sudan have resulted in numerous atrocities and destabilizing consequences for the
country.

Q 111. 30630558  Which of the following can be concluded from the information present in the passage?

a)  
The Rapid Support Forces (RSF) are a well-regarded paramilitary group that has contributed significantly to Sudan's
stability.
b)  
Gen. Dagalo's close ties with Russia's Wagner private military company and Saudi Arabia provide him with
substantial strategic advantages.
c)  
The civilian population in Sudan has been largely unaffected by the political turmoil and the power struggle between
the two generals.
d)  
The international community's calls for a truce have been successful in de-escalating tensions between the conflicting
parties in Sudan.
Direction for questions 106 to 135: Read the following passages and answer the questions that follow.

Passage – 2

Trade data released by the Ministry of Commerce and Industry on Thursday provides an indication of the headwinds
to growth. India's merchandise exports and imports have now declined in three of the last four months as tightening
financial conditions across the world have weighed down global and domestic demand. As per the latest data,
merchandise exports contracted by almost 14 per cent in March as global demand continued to weaken, while
imports fell by around 8 per cent pointing towards slowing domestic demand. Considering the slowdown in the global
economy this year - in its latest world economic outlook, the International Monetary Fund has pegged global growth at
2.8 per cent in 2023, down from 3.4 per cent in 2022 - exports are likely to come under further pressure in the months
ahead. While the full extent of the downturn will only be evident in the months ahead, considering the strong linkages
to the broader economy, a deep contraction in merchandise exports will weigh down the manufacturing sector in
India, acting as a drag on overall domestic economic activity.

At the aggregate level, merchandise exports rose to $447 billion in 2022-23, up 6 per cent from $422 billion in 2022-
23. But strip away non-oil exports, and exports in the full year were actually lower than the year before. While
electronic goods have registered a significant expansion, as have rice exports, major segments, including labour
intensive categories, such as gems and jewellery, cotton and man-made yarn, textiles as well as engineering goods,
have seen lacklustre performance. According to a report by Nomura, excluding oil and gems and jewellery, core
exports have declined for four straight months, with major markets such as the US and the EU witnessing a sharp
slowdown. Alongside the decline in exports, the fall in imports is equally disconcerting. Again excluding oil and gems
and jewellery, imports in March were lower than in the same period the year before. Weakness is being observed
across both investment and consumer goods sections which points to a slackening of domestic demand.

The latest data also shows that the merchandise trade deficit widened to $267 billion in 2022-23, from $191 billion in
2021-22. Considering that both exports and imports are likely to remain weak this year, to what extent domestic
demand fares better than external demand will determine the country's overall current account deficit. While some
analysts expect the deficit to be around levels seen last year, others are more pessimistic, and, even as the services
trade surplus has been strong, are penciling in a higher deficit level.
Q 112. 30630558  Which of the following, if true, would most weaken the author's arguments?

a)  
The global economy experiences a sudden and unexpected upswing, leading to increased demand for Indian exports.
b)  The Indian government implements protectionist policies to boost the domestic manufacturing sector.

c)  The decline in imports indicates a shift towards domestic production and increased self-reliance in India.

d)  
The Indian government successfully negotiates new trade agreements with major trading partners, improving export
prospects.

Q 113. 30630558  Which of the following, if true, would most strengthen the author's arguments?

a)  The International Monetary Fund further downgrades global growth projections for the current year.

b)  India's merchandise exports show a sudden surge, defying the global economic slowdown.

c)  The Indian government announces significant infrastructure investments to boost domestic demand.
d)  A report reveals that the decline in imports is a result of increased domestic production in India.
Q 114. 30630558  Which of the following accurately reflects the change in India's merchandise trade deficit between
2021-22 and 2022-23?

a)  The merchandise trade deficit decreased to $191 billion in 2022-23 from $267 billion in 2021-22.

b)  The merchandise trade deficit remained constant at $267 billion in both 2021-22 and 2022-23.
c)  The merchandise trade deficit increased to $267 billion in 2022-23 from $191 billion in 2021-22.

d)  The merchandise trade deficit increased to $447 billion in 2022-23 from $422 billion in 2021-22.

Q 115. 30630558  Which of the following is the author most likely to disagree with?

a)  The merchandise trade deficit has widened in 2022-23 compared to the previous financial year.

b)  India's merchandise exports and imports have been negatively affected by tightening financial conditions.

c)  Weakness in domestic demand is evident from the fall in imports of both investment and consumer goods.
d)  The slowdown in the global economy will have no impact on India's export performance.

Q 116. 30630558  Which of the following factors, if true, would have the most significant impact on India's current
account deficit?

a)  A decrease in oil prices b)  A decrease in domestic demand for investment goods

c)  An increase in global demand for rice exports d)  An increase in domestic demand for consumer goods

Q 117. 30630558  Which of the following can be concluded from the information present in the passage?

a)  The decline in India's merchandise exports is primarily due to a decrease in demand for oil.

b)  
The decline in India's merchandise exports is limited to labour-intensive categories such as gems and jewellery,
cotton and man-made yarn, and textiles.
c)  
The decline in India's merchandise exports is likely to have a negative impact on the country's manufacturing sector.
d)  The decline in India's merchandise exports is likely to be offset by a strong services trade surplus.

Direction for questions 106 to 135: Read the following passages and answer the questions that follow.

Passage – 3

Commemorating 50 years of Project Tiger - India's first species-centric, inclusive, conservation endeavour - recently
in Karnataka, the prime minister, Narendra Modi, underlined his government's steadfast adherence to the fine balance
between ecology and economy. Mr Modi has perhaps based his claim on the findings of the latest tiger census. The
quadrennial counting exercise revealed a rise in the minimum tiger count by 200 to 3,167. India now carries 75% of
the global tiger population. Fittingly, Mr Modi also announced the launch of the International Big Cats Alliance, a 97-
nation bloc that will focus on the conservation of seven big cat species in the wild - tiger, leopard, jaguar, lion, snow
leopard, cheetah and puma.

Yet, the success must not make conservation policy indifferent to the challenges ahead. The findings in the census
indicate several worrying anomalies. While the Shivalik range, Gangetic plains, and north-eastern hills have
witnessed substantial growth in big cat populations, local extinctions have been recorded from central India and the
Western Ghats, even though the latter is one of India's biodiversity hotspots. This can be attributed to the
fragmentation of forest cover and the loss of prey base. The tiger population is expected to touch 5,000 in the course
of this decade but most tiger reserves - the Sundarbans are an example - have reached their carrying capacity. This
only heightens the possibility of sustained man-animal conflict. Moreover, heavy concentration of the species in
specific sites also puts them at the risk of epidemics. The redistribution of tigers should be looked at. But this would
require the creation of forests that are suitable to sustain new populations. Given the shrinking of dense forest cover -
a manifestation of Mr Modi's government prioritising economy over ecology - the relocation of India's national animal
would encounter significant problems. Saving the tiger or, for that matter, any wild animal cannot be looked at in
isolation. It requires comprehensive, overlapping interventions involving the regeneration and protection of forests,
their inhabitants as well as communities that share a symbiotic bond with the wild. It is a massive, layered challenge.
India's conservation policy must be ready to meet it.
Q 118. 30630558  What is the passage's main conclusion regarding India's tiger conservation efforts?

a)  The increase in tiger population is the only factor that India's conservation policy should focus on.

b)  India's tiger conservation efforts have been completely successful, with no significant challenges remaining.
c)  
India's conservation policy must address the complex and layered challenges, including habitat fragmentation and
man-animal conflicts, through comprehensive interventions.
d)  
The International Big Cats Alliance alone will be sufficient to address all conservation challenges related to India's
tiger population.

Q 119. 30630558  Based on the passage, which inference can be drawn about the challenges faced in India's tiger
conservation efforts?

a)  India's conservation policy has successfully addressed all issues related to tiger population growth.

b)  The International Big Cats Alliance will solely focus on India's tiger conservation challenges.
c)  Habitat fragmentation and loss of prey base are the primary reasons behind local extinctions in certain regions.

d)  The government's focus on the economy has had no impact on the conservation of India's national animal.

Q 120. 30630558  Which of the following options would strengthen the author's argument about the need for
comprehensive, overlapping interventions in India's conservation policy?

a)  
Charting instances of successful man-animal conflict resolution in other countries that have implemented
conservation strategies.
b)  
Emphasizing the importance of increasing the global tiger population without addressing habitat or mananimal conflict
issues.
c)  
Focusing solely on the economic benefits of the tiger conservation initiatives implemented by the Indian government.
d)  Proposing a reduction in the scope of the International Big Cats Alliance to concentrate only on tiger conservation.

Q 121. 30630558  As a policymaker aiming to address the challenges highlighted in the passage, which of the
following strategies would most effectively align with the author's recommendation for comprehensive, overlapping
interventions in India's conservation policy?

a)  
Implementing a targeted approach to increase the tiger population in regions with high carrying capacities, while
disregarding habitat fragmentation.
b)  
Investing heavily in economic development projects within and around protected areas, without considering the
potential impact on wildlife conservation.
c)  
Developing a comprehensive plan to regenerate and protect forests, address man-animal conflicts, and support
communities sharing a symbiotic bond with wildlife.
d)  
Focusing exclusively on the prevention of epidemics within high-density tiger habitats without considering habitat
fragmentation or man-animal conflicts.

Q 122. 30630558  Which of the following external pieces of evidence would weaken the author's argument about the
need for comprehensive, overlapping interventions in India's conservation policy?

a)  A recent study showing that habitat fragmentation has had minimal impact on tiger populations in certain regions.
b)  
An international report highlighting the success of single-focus conservation efforts in preserving endangered species.
c)  
A case study demonstrating the positive effect of economic development on the overall well-being of local
communities around protected areas.
d)  
The discovery of a new, less fragmented habitat for the relocation of tigers, reducing the likelihood of man-animal
conflicts.

Q 123. 30630558  Based on the passage, which of the following assumptions underlies the author's argument for
comprehensive, overlapping interventions in India's conservation policy?

a)  The existing efforts to increase tiger populations are sufficient for long-term species survival.

b)  
The government's prioritization of economy over ecology has not caused any negative impact on conservation efforts.
c)  
Addressing multiple conservation challenges simultaneously would result in a more effective and sustainable
approach.
d)  
The International Big Cats Alliance is solely responsible for the protection and conservation of all seven big cat
species in the wild.

Direction for questions 106 to 135: Read the following passages and answer the questions that follow.

Passage – 4

The Government is reportedly contemplating an ambitious export target of $900 billion for the current fiscal, which will
be almost 17 per cent more than exports of $770 billion in 2022-23. Optimism in official circles is occasioned by
various factors, one of which is the World Trade Organisation's (WTO's) upwardly revised forecast for trade growth. It
estimates that global merchandise trade volume growth will be 1.7 per cent of world GDP in the calendar year 2023;
in October 2022, its estimate was 1 per cent. Second, the $94-billion increase in exports in 2022-23 despite huge
problems has made the officials upbeat. Third, they are confident that the Make in India, Make for World programme
and the Rs-2 lakh crore production linked incentive (PLI) schemes to incentivise exports are doing well, especially in
electronic exports. Electronics figured in the top 10 items of merchandise exports; it was 5.7 percent in its share in
2022-23 at $25.39 billion, up from 3.71 per cent in 2021-22. US technology giant Apple has augmented its iPhone
production in India substantially; it rose from 1 per cent of global production to 7 per cent in two years. Apple partners
like Foxconn Technology Group and Pegatron Corp are assembling their products in India. Chinese smartphone
maker Vivo will also invest Rs 1,100 crore in a new 169-acre manufacturing facility in Greater Noida. The Government
believes that the $900-billion exports target is not unrealistic but it will fix the target after wide consultations with all
stakeholders.

Policy and decision makers, however, would do well to focus more on merchandise exports. In the last fiscal, these
were $447.46 billion, up from $422 billion in 2021-22; this was a rise of merely 6 per cent. Services exports, on the
other hand, grew from $254.53 billion in 2021-22 to $322.72 billion in 2022-23, registering a rise of 26.8 per cent. In
other words, most of the rise in exports is because of the excellent performance of services. This is a familiar story, a
story that underlines the Indian economy's Achilles' heel-manufacturing. National Manufacturing Policy, which was
announced in 2011, aimed to increase the share of manufacturing in GDP to 25 percent by 2025. In 2021, it was 14
per cent, down from 15 per cent in 2015. In fact, since 1960 this share never touched even the 20 per cent mark. If
the Government really wants to increase merchandise exports, it has to ensure buoyancy in manufacturing. It is doing
a lot directly and indirectly to boost factory output. At the Central level, there are not many procedural rigours, but the
same cannot be said about the State level; it is here that industrialists, especially from micro, small and medium
enterprises or MSMEs, face problems. The Centre must coax, nudge or goad States to become more sensitive to the
concerns of MSMEs-and not just in hi-tech sectors like electronics. There cannot be a boom in merchandise exports
without a boom in manufacturing.
Q 124. 30630558  What is the main conclusion made by the author in the passage?

a)  The Indian government should focus on increasing service exports to achieve the ambitious $900 billion target.

b)  The $900 billion export target is unrealistic and cannot be achieved with the current policies in place.

c)  The government must Address MSME challenges and promote manufacturing to boost merchandise exports.

d)  
The WTO's upwardly revised trade growth forecast will automatically lead to an increase in India's merchandise
exports.

Q 125. 30630558  According to the passage, why did the Indian economy's merchandise exports grow by only 6% in
the last fiscal?

a)  
The National Manufacturing Policy's target of increasing manufacturing's share in GDP to 25% by 2025 has already
been achieved.
b)  
The government's focus on the Make in India and PLI schemes has led to decreased investment in the manufacturing
sector.
c)  There is low manufacturing sector share in GDP, services driving export growth.
d)  The Indian government has not taken any initiatives to improve the manufacturing sector's performance.

Q 126. 30630558  Which of the following would strengthen the author's argument that focusing on manufacturing is
crucial for increasing merchandise exports?

a)  The recent increase in service exports is primarily due to the growth of the IT and software industries.

b)  The manufacturing sector has shown a consistent increase in its share of GDP over the past decade.
c)  Countries with robust manufacturing sectors historically achieve higher merchandise export growth.

d)  The WTO's upward revision of trade growth forecasts is primarily due to the growth in the services sector.

Q 127. 30630558  Which of the following external pieces of evidence would weaken the author's argument that
focusing on manufacturing is crucial for increasing merchandise exports?

a)  India's service exports have consistently outperformed merchandise exports in terms of growth rate.

b)  Recent studies indicate high merchandise export growth rates without strong manufacturing.

c)  The Make in India program has led to increased foreign direct investment in the manufacturing sector.

d)  
States with a higher focus on manufacturing have experienced better export growth than states with a focus on
services.

Q 128. 30630558  Which of the following inferences can be drawn from the passage?

a)  The Indian government has not taken any initiatives to boost manufacturing in the country.

b)  The author believes that the $900-billion export target is unrealistic and unattainable.

c)  The Make in India and PLI schemes have contributed to the growth in electronic exports.

d)  
The WTO's upward revision of trade growth forecasts is solely responsible for the government's ambitious export
target.

Q 129. 30630558  Which of the following would the author of the passage disagree with?

a)  The Indian economy's Achilles' heel is its manufacturing sector.

b)  The growth in services exports is sufficient for achieving the ambitious $900-billion export target.
c)  The government needs to address challenges faced by MSMEs to increase merchandise exports.

d)  Apple's increased iPhone production in India has positively impacted electronic exports.

Direction for questions 106 to 135: Read the following passages and answer the questions that follow.

Passage – 5

The media are wondering why the public continues to be sour about the U.S. economy despite the Biden
Administration's attempt to explain how great it is. The answer lies in a startling comment by House Speaker Kevin
McCarthy on Monday in his speech at the New York Stock Exchange.

"Americans have received a pay cut for 24 consecutive months-the longest streak in American history-as inflation has
persisted," Mr. McCarthy said. "In fact, since President Biden took office, families have lost the equivalent of $7,400
worth of income."

If true, this means that Americans haven't had a raise since Mr. Biden took office. We've been wondering about that
ourselves, as we monitor the monthly inflation statistics and their impact on real earnings. The nearby chart tells the
story, and, sure enough, the Biden Presidency has been dreadful for incomes after inflation, which is one proxy for the
overall American standard of living.

The chart shows the trend in average weekly earnings for private workers who aren't supervisors in 1982-1984 dollars
from the Bureau of Labor Statistics. Weekly earnings hit a peak of $338.57 in January 2021, the last month of the
Trump Presidency.

They proceeded to fall in 2021, especially as inflation began to pick up. Average weekly earnings after inflation really
hit the skids starting in October 2021, and they reached a low of $322.96 last June as inflation peaked at 9.1%. They
have slowly climbed most months since and were up to $326.40 in March. Yet that's still a decline over two years of
3.6%.

This is ugly by any measure, and it documents the price that Americans are paying for inflation unleashed by the
spending and monetary excesses of President Biden, the last Democratic Congress and the Federal Reserve.
Americans understand this-which goes a long way to explaining why only 37.4% of the public approves of Mr. Biden's
performance on the economy in the Real Clear Politics polling average.

More Americans are working, but their standard of living isn't rising. Bidenomics has been a bust for the middle class
that Mr. Biden claims to champion.
Q 130. 30630558  Which of the following pieces of evidence from the passage strengthens the author's argument that
Bidenomics has been a bust for the middle class?

a)  The fact that more Americans are working under Biden's presidency.

b)  
The mention of a 37.4% approval rating for Biden's performance on the economy according to the Real Clear Politics
polling average.
c)  
The decrease in average weekly earnings for private workers who aren't supervisors since President Biden took
office.
d)  The peak in inflation at 9.1% in June.

Q 131. 30630558  Which of the following external pieces of evidence could weaken the author's argument that
Bidenomics has been a bust for the middle class?

a)  A report showing an increase in the number of small businesses opened during Biden's presidency.

b)  A study demonstrating a significant decrease in poverty rates during the Biden Administration.
c)  A statistic indicating a higher GDP growth rate under Biden compared to the previous administration.

d)  A survey revealing increased satisfaction with healthcare access during the Biden Administration.

Q 132. 30630558  Based on the passage, which inference can be made about the relationship between the Biden
Administration's economic policies and the public's perception of the economy?

a)  
The Biden Administration's economic policies have been effective in controlling inflation, but the public remains
skeptical due to the influence of the media.
b)  
The public's negative perception of the economy persists due to declining real earnings after inflation, despite the
administration's efforts to highlight improvements.
c)  
The public's perception of the economy is primarily influenced by political bias rather than the actual impact of the
Biden Administration's economic policies.
d)  
The Biden Administration's economic policies have led to an increase in unemployment, causing the public's negative
perception of the economy.

Q 133. 30630558  Based on the passage, which factor is primarily responsible for the decline in the American
standard of living under the Biden Administration?

a)  The lack of effective fiscal policies to boost employment opportunities.

b)  Real earnings have fallen due to rising inflation and monetary excesses.
c)  The Biden Administration's focus on environmental policies rather than economic growth.

d)  The inability of the Biden Administration to reduce income inequality.

Q 134. 30630558  According to the passage, which statement best explains the reason for the Biden Administration's
low approval ratings on economic performance?

a)  
There has been a lack of progress in dealing with income inequality under the Biden Administration, resulting in public
dissatisfaction.
b)  The Biden Administration's environmental policies have negatively impacted economic growth.
c)  The public's negative perception of the economy is primarily influenced by political bias.

d)  After inflation, American real earnings have declined under the Biden Administration, resulting in dissatisfaction.

Q 135. 30630558  Which assumption made by the author is necessary for his argument that the Biden
Administration's economic policies have negatively impacted the American standard of living?

a)  The decline in real earnings after inflation is a reliable measure of the American standard of living.
b)  The increase in inflation is primarily due to the Biden Administration's excessive spending policies.

c)  The public's negative perception of the economy is solely based on the decline in real earnings after inflation.

d)  
The decline in real earnings after inflation is not related to any external factors beyond the Biden Administration's
control.

Quantitative Techniques
Directions for questions 136 to 140: Answer the questions on the basis of the information given below.

The sample population 300 individuals were chosen for the study. Among the 300 respondents, 175 were males and
125 females.

For establishing a relationship between junk food consumption and residential status three questions were asked: Are
you a permanent resident of Delhi? Percentage of income spent on junk foods? How many times you take junk food
during a week time? Out of 300 students surveyed by us 60% respondents were non Delhiites and 40% were
Delhiites. 45% of non-Delhiites take junk food more than or equal to thrice a week, 50% of them take junk food twice
a week and only 5% of them take junk food once a week. In case of Delhiites , 10% of them take junk food more than
or equal to thrice a week, 30% of them take junk food twice a week and 60% of them take junk food once a week.
However, if we talk in terms of finance available and frequency of junk food consumption, we found that 40% of non-
Delhiites spent more than Rs. 3,000 in a month over junk foods as compared with Delhiites in case of which only 20%
of them spent more than Rs. 3,000 a month over junk food consumption.

[Source: https://journals.indexcopernicus.com/]

[Note: values have been approximated to the nearest integer values]


Q 136. 30630558  When the students were asked to select the factor that influenced them in selecting the type of the
junk food, 41% opted for taste, 35% opted for ease in availability and less time consuming, 13% said they are
influenced by advertisements and only 11% accepted it as a part of their lifestyle. If the number of males in each of
the mentioned factors are in the ratio 3:2:1:1 respectively, then what is the ratio of number of females who selected
taste and who selected influence by advertisements as the factors that influenced them?

a)  24 : 7 b)  12 : 5 c)  16 : 7 d)  24 : 5

Q 137. 30630558  Which of the following statements is true?

a)  
The number of Delhiites who take junk food once a week is greater than non-Delhiites who take junk food more than
or equal to thrice a week.
b)  The ratio of non-Delhiites to Delhiites who take junk food twice a week is 5:3.
c)  
The number of Delhiites who take junk food twice a week is 4 times the number of non-Delhiites who take junk food
once a week.
d)  None of the statements is true.

Q 138. 30630558  Among Delhiites, 45% of the students considered junk food as addictive, 25% considered junk food
as non-addictive while 30% were unable to answer. 50% of students who considered junk food as addictive have said
they were completely aware of the chemicals present in junk food and remaining were unaware. Similarly, 2/3rd of
students who considered junk food as non-addictive were aware of the chemicals present in junk food and remaining
were unaware. What approximate percent of Delhiites were aware of the chemicals present in Junk food?

a)  35% b)  43% c)  39% d)  45%

Q 139. 30630558  Of non-Delhiites, 40% of females spend more than Rs. 3,000 on Junk food. If 75 males do not
spend more than Rs. 3,000, then number of male non-Delhiites who spend more than Rs. 3,000 is what percent of
number of female non-Delhiites?

a)  between 95% and 100% b)  between 80% and 85% c)  Less than 90% d)  Greater than 90%

Q 140. 30630558  Of Delhiites, number of students who spend more than Rs. 3,000 on junk food, spends on an
average 80% of their pocket money on Junk food. The ratio of these Delhiite students who spent nearly Rs. 6,000,
Rs. 4,000 and Rs. 3,500 per student on Junk food is in the ratio 1 : 2 : 3 respectively. What is the approximate
average amount (in Rs.) of pocket money of these students?

a)  5104.17 b)  5421.33 c)  5905.19 d)  6020.20

Directions for questions 141 to 145: Answer the questions on the basis of the information given below.
Q 141. 30630558  What is the difference between the number of males aspiring to be Engineers and Lawyers
together and the number of females aspiring for the same professions?

a)  24 b)  21 c)  25 d)  18

Q 142. 30630558  The total number of students aspiring to be Architects is approximately what percent of the total
number of students aspiring to be Doctors?

a)  45.28% b)  58.35% c)  60.54% d)  52.38%

Q 143. 30630558  What is the difference between the total number of male and female students in the survey?

a)  1284 b)  1134 c)  1054 d)  1164

Q 144. 30630558  The number of females aspiring to be Doctors is what percentage of the number of females
aspiring to be Teachers?

a)  42% b)  28% c)  15% d)  35%

Q 145. 30630558  What is the ratio of the number of males aspiring to be Architects to the number of males aspiring
to be Teachers?

a)  11 : 5 b)  3 : 2 c)  5 : 11 d)  2 : 3

Directions for questions 146 to 150: Answer the questions on the basis of the information given below.

Minimum Support Price (MSP) is a form of market intervention by the Government of India to insure agricultural
producers against any sharp fall in farm prices. The minimum support price for a crop is announced by the
Government of India at the beginning of the sowing season on the basis of the recommendations of the Commission
for Agricultural Costs and Prices (CACP). MSP is price fixed by the Government of India to protect the farmers against
excessive fall in price during bumper production years. The minimum support price provides a guaranteed return of at
least 1.5 times the cost of production.

During 2022-23, the MSP for Jute was fixed at Rs. 4,736/- per quintal, which provided returns of 60% over the cost of
production (COP) per quintal for 2022-23. In 2021-22 the COP was 10% lower than that in 2022-23 whereas the MSP
was 75% above the cost. In 2020-21 the MSP and the COP were Rs.162 less and Rs.136 more than the respective
values in 2021-22. The COP in 2022-23 was 18.4% more than the COP of 2019-20 whereas the MSP in 2020-21 was
12.5% more than that in 2019-20. In 2018 - 19 the MSP was 65% more than the COP in the same year, which was
Rs.100 less than that in 2019-20.
Q 146. 30630558  What was the ratio of the MSP of Jute in the years 2020-21 and 2018-19 respectively?

a)  20 : 17 b)  24 : 23 c)  25 : 22 d)  22 : 19

Q 147. 30630558  What was the approximate percentage returns on the average cost of production of Jute during the
year 2020-21?

a)  72.61% b)  55.81% c)  65.63% d)  60.71%

Q 148. 30630558  What was the percentage increase in Cost of production of Jute from 2018-19 to 2022-23?

a)  

b)  

c)  28%
d)  32%

Q 149. 30630558  During which two years did the farmers witness the same percentage returns for their Jute
produce?

a)  2022-23 and 2019-20 b)  2018-19 and 2021-22 c)  2021-22 and 2020-21 d)  2022-23 and 2020-21

Q 150. 30630558  What was the average cost of production per quintal of Jute from 2018-19 to 2022-23?

a)  Rs.2,446.20 b)  Rs.2,554.40 c)  Rs.2,664.80 d)  Rs.2,284.00

You might also like